#18 Rosh Review

Ace your homework & exams now with Quizwiz!

Question: In a patient with AIDS, what medication is given prophylactically at a CD4 count of less than 100 cells/μL to prevent infection with toxoplasmosis gondii?

Answer: Trimethoprim/sulfamethoxazole Rapid Review Pneumocystis Pneumonia (PCP) Patient with a history of HIV Complaining of gradual onset of non-productive cough Labs will show CD4 < 200, increased LDH CXR will show bilateral infiltrates (bat wing pattern) Most commonly caused by Pneumocystis jirovecii Treatment is TMP-SMX

Question: What medications are recommended for treatment of H. pylori associated peptic ulcer disease?

Answer: Triple therapy with clarithromycin, amoxicillin (or metronidazole) and a PPI for 10-14 days. Rapid Review Gastritis Acute: NSAIDs > alcohol Type A chronic: pernicious anemia Type B chronic : H. pylori

Question: True or false: Malaria infection must be reported to the state health department?

Answer: True.

Question: What is the mortality rate of thyroid storm?

Answer: Up to 75%. Rapid Review Hyperthyroidism Patient will be complaining of heat intolerance, palpitations, weight loss, tachycardia, and anxiety PE will show hyperreflexia Labs will show low TSH and high free T4 Most commonly caused by Graves disease (autoimmune against TSH receptor) Treatment is methimazole or PTU Comments: Propylthiouracil (PTU) P for pregnant

Question: How many patients with depression meet criteria for a bipolar disorder?

Answer: Between 21 and 26%. Rapid Review Bipolar Disorder Lifelong, extreme mood swings Mania, hypomania Major depressive disorder Inflated self esteem Decreased need for sleep Pressured speech Flight of ideas Excessive pleasurable activity

Question: How do you calculate body mass index (BMI)?

Answer: Body Mass Index (BMI) is calculated by dividing a person's weight (kg) by their height (in meters squared). BMI is a fairly reliable indicator of body fatness for most people.

Question: What is the major adverse effect of zidovudine?

Answer: Bone marrow suppression.

Question: What is the treatment for diverticulitis?

Answer: Bowel rest, intravenous fluid hydration, broad-spectrum antibiotics. Rapid Review Diverticulitis Patient will be complaining of abdominal pain that is localized to the left lower quadrant, fever, nausea, vomiting, and a change in bowel habits PE will show localized guarding, rigidity, and rebound tenderness Diagnosis is made by CT scan Treatment is abx

Question: What is congenital adrenal hyperplasia?

Answer: An autosomal recessive disorder that causes a defect in the 21-hydroxylase enzyme causing defective glucocorticoid biosynthesis and adrenal gland androgen overproduction. Rapid Review Adrenal Neoplasm Types: functioning, nonfunctioning Rx nonfunctioning: resect if > 4 cm Rx functioning: treat disorder

Question: What drugs should be avoided in patients taking dopaminergic therapy?

Answer: Antipsychotic drugs because they can precipitate akathisia or tardive dyskinesia (with prolonged use). Rapid Review Parkinson's Disease Lewy bodies, substantia nigra dopaminergic neuron loss TRAP: Tremor (resting, "pill rolling"), Rigidity, Akinesia, Postural instability Carbidopa/levodopa, anticholinergic drugs Avoid antipsychotics

Question: What percentage of women with preexisting hypertension develops preeclampsia during pregnancy?

Answer: Approximately 20%.

Question: What is the most common sequela of mitral stenosis?

Answer: Atrial fibrillation. Increased atrial irritability and hypercoagulability associated with pregnancy combined with increased left atrial pressure, increase the risk of atrial fibrillation and left atrial thrombus formation. Rapid Review Rheumatic Fever Patient with a history of GAS infection Complaining of fever, red skin lesions on the trunk and proximal extremities, and small, non-tender lumps located over the joints PE will show JONES criteria: Joints, Oh, no carditis!, Nodules, Erythema marginatum, Sydenham's chorea Labs will show anti-streptolysin O, anti-DNase B, positive throat culture, or positive rapid antigen test Treatment is antibiotics, NSAIDs Comments: Modified Jones Criteria for a first episode of acute rheumatic fever: need 2 major or 1 major and 2 minor

Question: What is the treatment for Chlamydia?

Answer: Azithromycin 1 gram x 1 or doxycycline 100 mg 1 tablet twice daily for 1 week.

Question: What finding on wet mount is diagnostic of bacterial vaginosis?

Answer: Clue cells (epithelial cells surrounded by bacteria). Rapid Review Trichomoniasis Patient will be a woman complaining of malodorous vaginal discharge PE will show frothy, green/yellow discharge, "Strawberry cervix" Labs will show pH >5, flagellated, motile, pear shaped Diagnosis is made by wet mount Most commonly caused by Trichomonas vaginalis Treatment is metronidazole

Question: When urine is collected by catheterization, what colony count is considered diagnostic for infection?

Answer: Colony count >10,000 cfu/mL. Rapid Review Pediatric Urinary Tract Infection Boys ≤ 1 year, girls < 2 Infants: GBS >1 year: E. coli High risk pts: US for VUR VUR → kidney damage

Question: Initially, how should cutaneous lupus be treated?

Answer: Cutaneous lupus should initially be treated with topical corticosteroids and avoidance of precipitating factors, such as sun exposure. Rapid Review Systemic Lupus Erythematosus (SLE) African-Americans, females Malar rash Arthritis Renal disease Cardiac: fibrinous pericarditis, Libman-Sacks endocarditis CNS: HA, stroke, seizures Drug induced: Hydralazine, INH, Procainamide, Phenytoin, Sulfonamides (HIPPS) Antinuclear antibodies (ANA): 100% sensitive, not specific Anti-dsDNA antibodies: 100% specific, poor prognosis Anti-Smith antibodies: 100% specific, not prognostic Antihistone antibodies: sensitive for drug-induced lupus NSAIDs, steroids, immunosuppressants, hydroxychloroquine

Question: What blood vessel is most commonly affected in mesenteric arterial embolism?

Answer: The superior mesenteric artery (SMA) is the most common site of arterial emoblism. Rapid Review Mesenteric Ischemia Risk factors: dysrhythmias (AF), recent MI, CAD Abdominal pain out of proportion to exam Lactic acidosis Dx: CT, angiography (gold standard)

Question: What is the most common long bone fracture?

Answer: Tibia.

Question: What are some anticholinergic side effects?

Answer: Dry mouth, blurred vision, constipation, drowsiness, sedation and urinary retention.

Question: What bacteria is associated with outbreaks of ground beef related hemorrhagic colitis?

Answer: E. coli O157:H7 Rapid Review Food Poisoning B. cereus: reheated rice C. botulinum: canned food C. perfinengens: reheated meat Ciguatera: sea bass, grouper, red snapper E. coli O157:H7: undercooked meat Salmonella: poultry, meat, eggs S. aureus: meats, mayonaiise, custard Scombroid: peppery tasting fish V. parahaemolyticus, V. vulnificus: seafood Yersinia: undercooked pork

Question: What is the most common cause of pediatric hypertension?

Answer: Essential, or primary, hypertension is most common in both adults and pediatrics. Rapid Review Coarctation of the Aorta PE will show higher blood pressure in the arms than in the legs EKG will show LVH CXR will show notching of ribs Diagnosis is made by echo Treatment is balloon angioplasty with stent placement, or surgical correction Comments: Associated with Turner's syndrome

Question: How often should patients be screened for diabetes mellitus?

Answer: Every three years.

Question: True or false: The majority of individuals seek professional intervention after the loss of a loved one?

Answer: False.

Question: What is the most frequent complaint in patients with chronic hepatitis C infection?

Answer: Fatigue.

Question: What is the classic triad of bacterial meningitis?

Answer: Fever, altered mental status, nuchal rigidity. Rapid Review Bacterial Meningitis (Adult) Streptococcus pneumoniae (most common) Headache, neck stiffness, photophobia, phonophobia, fever Meningismus, jolt accentuation, Brudzinski's sign (flex neck), Kernig's sign (extend knees) Acute: (< 24 hrs) bacterial Subacute: (1-7 days) lymphocytic/viral Chronic: (>1 week) insidious fungal (Cryptococcus in HIV patients) Empiric ABX before CT/LP Steroids Prophylaxis (rifampin) of close contacts (N. meningitidis and H. influenzae) Empiric antibiotic by age: 18 - 50 years: ceftriaxone + vancomycin > 50 years: ceftriaxone + ampicillin + vancomycin

Question: What are other head and neck physical signs of hyperthyroidism?

Answer: Fine, thin, moist skin, prominent eyes with lid retraction (proptosis, or exophthalmos) and a startled expression. Rapid Review Hyperthyroidism Patient will be complaining of heat intolerance, palpitations, weightloss, tachycardia, and anxiety PE will show hyperreflexia Labs will show low TSH and high free T4 Most commonly caused by Graves disease (autoimmune against TSH receptor) Treatment is methimazole or PTU Comments: Propylthiouracil (PTU) P for pregnant

Question: When is surgery recommended for treating an orbital blow-out fracture?

Answer: Fracture >50% of the floor, extraoccular muscle entrapment, enophthalmos or diplopia. Rapid Review Orbital Blowout Fracture Patient with a history of direct trauma to the orbit Complaining of limitation of upward gaze, infraorbital anesthesia PE will show inferior rectus entrapment, enophthalmos Xray will show teardrop sign Diagnosis is made by CT Treatment is surgery

Question: Genital herpes increases the risk for acquiring what other sexually transmitted disease?

Answer: HIV infection. Rapid Review Herpes Simplex HSV-1: oral, keratitis HSV-2: genital, neonatal Grouped lesions on an erythematous base Multinucleated giant cells Most common cause of encephalitis HSV keratitis: dendrites on fluorescein scan Rx: acyclovir

Question: What is the first-line therapy for recurrent endometrial carcinoma?

Answer: High dose progestin preparations. Rapid Review Endometrial Cancer Patient will be a postmenopausal woman Complaining of abnormal vaginal bleeding Diagnosis is made by transvaginal ultrasound or endometrial biopsy Most common type is adenocarcinoma Treatment is total abdominal hysterectomy and bilateral salpingo-oophorectomy (TAH-BSO)

Question: Is the lactate dehydrogenase level high or low in an exudate pleural fluid?

Answer: High.

Question: How long after treatment with thiamine should the symptoms of ataxia and confusion begin to resolve?

Answer: Improvement of these symptoms can take days to weeks but will not improve in many patients despite appropriate therapy. Rapid Review Wernicke-Korsakoff Syndrome Wernicke (ophthalmoplegia + confusion + ataxia) Korsakoff (irreversible memory loss) Thiamine (B1) deficiency Alcoholic Patient Rx: thiamine

Question: What is the most commonly occurring hernia?

Answer: Indirect inguinal hernia. Rapid Review Small Bowel Obstruction Causes: adhesions (pelvic surgery) > tumor > hernia (inguinal) Proximal = bilious vomiting, distal = feculent vomiting High pitched bowel sounds X-ray: dilated bowel, air fluid levels, "stack of coins" or "string of pearls" sign NGT, surgery

Question: Which anemia often precedes lead toxicity?

Answer: Iron deficiency anemia due to its association with pica.

Question: What happens to plasma creatinine concentration in pregnancy?

Answer: It decreases.

Question: How long after cessation of alcohol ingestion does delirium tremens occur?

Answer: It typically occurs at least 72 hours after cessation of alcohol ingestion. Rapid Review Ethanol Withdrawal Autonomic hyperactivity (↑ HR, ↑ BP, diaphoresis) Visual, tactile hallucinations Seizures Delirium tremens: autonomic hyperactivity, psychosis, peaks 2-5 days after cessation Tremulousness → hallucinations → seizures → delirium tremens Rx: BZDs

Question: What is the role of topical acyclovir in herpetic whitlow?

Answer: Topical acyclovir may reduce the length of symptoms. Rapid Review Herpetic Whitlow Clear painful vesicles Do not I&D Acyclovir

Question: What is the first line medical therapy for Crohn's disease?

Answer: Mesalamine (ileitis) or sulfasalazine (ileocolitis/colitis) Rapid Review Crohns Disease Involvement: mouth to anus but usually distal ileum, colon, spares rectum Transmural inflammation Bloody diarrhea uncommon Exam: aphthous ulcers, anal fissures, perirectal abscesses, anorectal fistulas Colonoscopy: skip lesions, cobblestone mucosa ASCA positive, p-ANCA negative No surgery Ileitis rx: mesalamine Ileocolitis/colitis: mesalamine or sulfasalazine

Question: Which medication reverses opioid-induced respiratory depression?

Answer: Naloxone.

Question: Young children with nasal polyp formation should be screened for which hereditary disorder?

Answer: Nasal polyp formation in a child is concerning for hereditary cystic fibrosis. Rapid Review Allergic Rhinitis Patient with a history of asthma, atopic dermatitis and sinusitis Complaining of sneezing, rhinorrhea, and nasal congestion PE will show infraorbital edema and darkening, transverse nasal crease, cobblestoning of the posterior pharynx Labs will show elevated serum IgE Treatment is glucocorticoid nasal spray Comments: nasal polyps, asthma, and aspirin-sensitivity (Samter's triad)

Question: Does excision or incision of a thrombosed hemorrhoid have a lower recurrence rate?

Answer: Office surgical excision results in a lower recurrence rate and earlier resolution of symptoms. Simple incision of the thrombosed hemorrhoid may not remove multiple clots and can result in higher recurrence rate. Rapid Review Hemorrhoids Internal Proximal to dentate line Painless bleeding External Below dentate line Visible Painful Initial rx: WASH (Water (sitz bath), Analgesics, Stool softeners, High fiber diet) Thrombosed hemorrhoid rx: excision with elliptical incision

Question: What are the classic symptoms of active tuberculosis?

Answer: Persistent malaise, anorexia, weight loss, fever, night sweats, and a chronic cough are classic symptoms.

Question: What is the most common cause of secondary amenorrhea?

Answer: Pregnancy. Rapid Review Primary Amenorrhea Definition: lack of menarche by 16 with normal 2° sexual characteristics or by 14 with no 2° sexual characteristics Etiologies: Hypothalamic/pituitary Ovarian Uterine Pseudohermaphroditism Lab workup: FSH, LH, prolactin, TFTs, testosterone, hCG Rx: treat underlying cause

Question: What are two absolute contraindication to treatment with clozapine?

Answer: Prior history of clozapine-induced agranulocytosis or myocarditis. Rapid Review Clozapine Agranulocytosis Seizures Myocardits Increased mortality in elderly patients with dementia-related psychosis

Question: What is the preferred route of medication administration if intravenous or intramuscular access is not available for a seizing child?

Answer: Rectal; diazepam is the agent of choice for this purpose. Rapid Review Febrile Seizure - Simple Patient will be a child 6 mos - 5 yrs Complaining of a single generalized seizure lasting < 15 mins Most commonly caused by rapid rise in temperature Treatment is supportive care

Question: What are the most common virus etiologies of acute rhinosinusitis?

Answer: Rhinovirus, influenza virus, parainfluenza virus and adenovirus. Rapid Review Sinusitis MCC: viral URI Pain over sinus Purulent rhinorrhea ABX for refractory cases

Question: The athlete who suffers multiple concussions and returns to play too soon is at risk for what syndrome?

Answer: Second impact syndrome. Rapid Review Concussion Brief LOC, amnesia No focal neurologic deficits Negative CT scan

Question: What are the treatments of choice for Osgood-Schlatter disease and Patellar tendonitis?

Answer: Self-limited and can be treated with rest, ice and NSAIDs. Rapid Review Osteosarcoma Most common malignant bone tumor 10-20 years old, > 65 years old Pain, swelling at tumor site X-ray: Codman's triangle, sunburst pattern Long bone metaphyses

Question: What is the preferred treatment of choice for a patient with acute bronchitis?

Answer: Since antibiotics are not indicated in the treatment of acute bronchitis, most cases are typically targeted toward symptomatic relief.

Question: For what other lung problem are cystic fibrosis patients at risk?

Answer: Spontaneous pneumothorax. Rapid Review Cystic Fibrosis Patient with a history of multiple recurrent respiratory infections or failure to thrive Diagnosis is made by elevated quantitative sweat chloride test Most commonly caused by autosomal recessive disorder in CFTR gene that results in the abnormal production of mucus Comments: Pseudomonas aeruginosa most common infecting bacteria

Question: What are the most common species responsible for bacterial conjunctivitis?

Answer: Staphylococcal and streptococcal species. Rapid Review Bacterial Conjunctivitis Patient will be complaining of red/pink eye with discharge usually worse in the morning PE will show purulent (yellow) discharge and crusting Most commonly caused by Staph aureus, Strep pneumoniae, H.Flu. Contact lens wearers: Pseudomonas aeruginosa Treatment is topical antibiotic drops

Question: What is the recommended treatment for Tularemia?

Answer: Streptomycin. Rapid Review Tularemia Francisella tularensis Tick borne illness Handling of infected animals: rabbits, rodents Hunters, butchers, cooks, campers Ulcer at site, lymphadenopathy, conjunctivitis, pneumonia Streptomycin

Question: What location of lymphadenopathy is most concerning for malignancy?

Answer: Supraclavicular. Rapid Review Lymphadenitis Patient will be a child PE will show cervical lymphadenitis that is tender, warm, and swollen Most commonly caused by Staphylococcus aureus and Streptococcus pyogenes

Question: Which medications are considered risk-reducing medications for women who are at an increased risk of breast cancer?

Answer: Tamoxifen & raloxifene. Inclusion criteria: Women who are asymptomatic, > 35 years old, and who are at increased risk of breast cancer, negative history for DVT and low risk for adverse medication side effects.

Question: What diagnosis should be suspected in a patient older than 50 years of age who has an ESR greater than 50?

Answer: Temporal (Giant cell) arteritis. Rapid Review Temporal Arteritis Elderly females Unilateral temporal HA, jaw claudication Ophthalmic artery occlusion → irreversible blindness Associated with PMR ↑ ESR, ↑ CRP Dx: temporal artery biopsy Immediate high-dose steroids

Question: What is the cause of struvite renal stones?

Answer: Urease producing bacteria like Proteus, Klebsiella and Staphylococcus. Rapid Review Nephrolithiasis Patient will be complaining of flank pain radiating to groin PE will show a patient that won't lay still and hematuria Diagnosis is made by helical CT Most commonly caused by calcium oxalate Struvite: staghorn calculi, urease producing bacteria Uric acid: radiolUcent on xray, gout Cystine: children with metabolic diseases Most common location is the ureterovesiclular junction (UVJ) Treatment is: < 5 mm: likely to pass spontaneously > 8 mm: unlikely to pass, lithotripsy

Question: What are the common cardiac catheterization findings associated with restrictive cardiomyopathy?

Answer: Ventricular "dip and plateau" pressures, and atrial M and W waves. Rapid Review Restrictive Cardiomyopathy Least common cardiomyopathy Most common cause: amyloidosis Impaired diastolic filling + preserved systolic function Right-sided heart failure symptoms ECG: low voltage, nonspecific changes

Question: What is the most common gynecological condition in prepubertal children?

Answer: Vulvovaginitis. Rapid Review Pediatric Vaginal Foreign Body Foul smelling or bloody discharge Exam indicated Consider abuse Use of a nasal speculum, procedural sedation, or trained assistant

Question: What public health achievement significantly contributed to the decline in the rate of dental caries?

Answer: Water flouridation.

Question: What is the phenomena of "core temperature after-drop" in frostbite management?

Answer: When large areas are rewarmed, vasoconstriction is relieved and cold, hyperkalemic and acidic blood returns to the central circulation. Rapid Review Frostbite Do not rewarm if possibility of refreezing Rx: address hypothermia first, rapid rewarming of affected part in 37-39°C water

Question: Can the alopecia associated with a kerion become permanent?

Answer: Yes, if left untreated, a kerion can lead to permanent alopecia.

Which of the following is recommended to use as an anti-hypertensive in a pregnant woman? Alpha-methyldopa (Aldomet®) Lisinopril (Zestril®) Losartan (Cozaar®) Valsartan (Diovan®)

Correct Answer ( A ) Explanation: Alpha-methyldopa is a centrally acting alpha-adrenergic medication used as a first line antihypertensive agent for pregnant women. The initial starting dose is 250 mg every 8 hours. This dose can be increased to 2 g/day as needed. If adequate control cannot be obtained, a second drug, most often nifedipine or hydralazine, can be added. Labetalol is gaining popularity for use as first-line single-agent therapy, but may be associated with fetal growth restriction. Most women with chronic hypertension will present at their first prenatal visit with the diagnosis. One third of pregnant women with essential hypertension will become normotensive in the first half of pregnancy. Often, their antihypertensive medications can be discontinued and restarted with the return of hypertension, typically after 28 weeks. The use of ACE inhibitors (B) and angiotensin II receptor antagonists (C and D) in late pregnancy has been associated with renal dysfunction, oligohydramnios, neonatal anuria, and other congenital anomalies such as skull ossification defects. However, data have also suggested an increased risk of congenital anomaly after exposure limited to the first trimester of pregnancy.

A 53-year-old man comes to the emergency department complaining of a 5-day history of a cough and shortness of breath. His temperature is 37.6°C (99.8°F). Auscultation of the lungs shows rhonchi and wheezing. Chest X-ray shows thickening of the bronchial walls in both lower lobes. Laboratory studies show a slightly elevated white blood count. Which of the following is the most likely diagnosis? Acute bronchitis Asthma Influenza Pneumonia

Correct Answer ( A ) Explanation: Based on the constellation of findings, this patient most likely has acute bronchitis. Acute bronchitis is a self-limited inflammation of the bronchi and typically presents with mucopurulent cough for more than 5 days. Chest X-ray will show thickening of the bronchial walls in the lower lobes and auscultation of the lungs usually reveals wheezing and rhonchi. Acute bronchitis is typically caused by viruses (eg, respiratory syncytial virus, rhinovirus, influenza), therefore, antibiotics are not indicated. Most cases of acute bronchitis are self-limited and resolve on their own within a few weeks. Although influenza (C) can cause acute bronchitis, it is not likely this patient has influenza. Influenza typically presents with cough, fever, sputum production, and constitutional symptoms. Pneumonia (D) presents similarly to influenza. Radiography typically shows lung consolidation (ie, lobar pneumonia). It is unlikely this patient has influenza or pneumonia due to the lack of fever and other constitutional symptoms. New onset asthma (B) is not likely in this patient because it is less frequent in older adults and usually presents before the age of 7.

A patient with major depression spends $100,000 at a casino over the past week. He presents with his wife, who states they now are in severe debt and cannot make any more mortgage payments. She also reports that over the past week her husband talked all the time and never slept more than 3 hours at a time. Which of the following is the most likely diagnosis? Bipolar I Bipolar II Hypomanic episode Major depression, acute flare

Correct Answer ( A ) Explanation: Bipolar disorder is characterized by manic or hypomanic behaviors that are sometimes accompanied by a depressive disorder. Onset is usually in late childhood and early adolescence, and there is no sex or race predilection. Bipolar type I is defined as at least 1 manic episode with or without major depression. Bipolar type II is defined as at least 1 hypomanic episode with a major depressive episode. A manic episode is defined as at least 1 week of at least 3 of the following findings that cause a marked occupational or social disturbance or an unequivocal uncharacteristic change in functioning or necessitates hospitalization: inflated self-esteem/grandiosity, decreased need for sleep, more talkative, flight of ideas/racing thoughts, distractibility, increased goal-directed activity or psychomotor agitation, excessive involvement in pleasurable activities that likely having poor repercussions. Patients with bipolar disorders have higher rates of general medical conditions and other mental health disorders. First-line treatment involves mood stabilizers with lithium, valproic acid, carbamazepine, lamotrigine, or atypical antipsychotics. Mood stabilizers treat at least one phase of bipolar disorder (mania or depression) without worsening the other phase. Antidepressants are not mood stabilizers and can precipitate mania. Behavioral therapy is also useful in the treatment of bipolar disorder. The symptoms of a manic episode of bipolar disorder are expressed in the mnemonic DIG FAST. Bipolar II (B) disorder is a hypomanic episode in a patient with depression without any history of any manic episodes. The above actions clearly indicate a manic, and not hypomanic, episode. A hypomanic episode (C) is defined as at least 3 of the manic episode symptoms that lasts at least 4 days and is not severe enough to cause marked social or occupational functioning or hospitalization. Not being able to pay a mortgage is a marked social disturbance. Major depression (D) is defined as five or more of the following, with at least one of the five being either depressed mood or loss of interest/pleasure, present during the same 2-week period: depressed mood most of the day, marked decreased interest/pleasure in most daily activities, weight loss when not trying to lose weight, insomnia or hypersomnia, psychomotor agitation or retardation, fatigue/loss of energy, feelings of worthlessness or excessive guilt, decreased concentration or indecisiveness and recurrent thoughts of death or recurrent suicidal ideation or suicide attempt or plan for suicide. This patient's actions do not match an acute flare of depression.

A 36-year-old man presents to your office for a physical exam. He has a family history of diabetes and his body mass index is 30. Which of the following provides the best screening for diabetes mellitus? Fasting plasma glucose Fingerstick testing of blood glucose Identification of acanthosis nigricans Urine glucose

Correct Answer ( A ) Explanation: Diabetes mellitus is a major cause of early death and illness in the United States and worldwide. Patient risk factors should be reviewed and screening should be done on patients at risk. Risk factors include diabetes in a first-degree relative, a body mass index (BMI) greater than 25, age greater than 45 years, a sedentary lifestyle, hypertension and hyperlipidemia. Screening may be done by measuring fasting plasma glucose (FPG), glycated hemoglobin (A1C), or two-hour plasma glucose with an oral glucose tolerance test. Three categories of results can occur with these screening tests: normal, increased risk of diabetes or "pre-diabetes" and diabetes mellitus. A diagnosis of diabetes mellitus requires a repeat of the same test with both showing abnormally high results. A FPG greater than 126 mg/dL on two occasions confirms the diagnosis of diabetes mellitus. Fingerstick testing of blood glucose (B) is used as a self-monitoring tool for patients with diabetes and does not have a role in diagnosis. Any patients with risk factors for diabetes should be screened. Acanthosis nigricans (C) is characterized by hyperpigmentation of the skin and is associated with various conditions such as obesity and various endocrinopathies such as acromegaly, PCOS, Cushing's disease, and insulin-resistant diabetes. However, it is a poor screening test for diabetes. Urine glucose (D) is not recommended in the screening of diabetes mellitus due to the insensitivity of the test.

A patient with acute pancreatitis is noted to have a pleural effusion on chest radiography. Which of the following findings would you expect to find on pleural fluid analysis? Elevated amylase concentrations Elevated triglyceride concentrations Low glucose concentrations Low pH of pleural fluid

Correct Answer ( A ) Explanation: Elevated amylase concentrations would be expected on pleural fluid analysis in the setting of acute pancreatitis. Although not routinely tested in pleural fluid samples, amylase measurements can assist when a pleural effusion is caused by a pancreatic or esophageal etiology. The finding of an amylase-rich pleural effusion, defined as either a pleural fluid amylase greater than the upper limits of normal for serum amylase or a pleural fluid to serum amylase ratio greater than 1.0, narrows the differential diagnosis of an exudative effusion to the following major possibilities: acute pancreatitis, chronic pancreatic pleural effusion, esophageal rupture and malignancy. Elevated triglyceride concentrations (B) on pleural fluid analysis supports the diagnosis of chylothorax or thoracic duct rupture, not acute pancreatitis. Low glucose concentrations (C) narrows the differential diagnosis of the exudate to rheumatoid pleurisy, complicated parapneumonic effusion or empyema, malignant effusion, tuberculous pleurisy, lupus pleurisy, or esophageal rupture. A low pH of pleural fluid (D) with a normal arterial blood pH is associated the same diagnoses associated with low pleural fluid glucose concentrations.

A 55-year-old man presents with amaurosis fugax, headaches, scalp tenderness, jaw claudication, occasional ear pain, malaise, and intermittent fevers. On exam, you note tenderness of his left temple. Laboratory studies reveal an erythrocyte sedimentation rate of 85. Which of the following is the most likely diagnosis? Giant cell arteritis Multiple sclerosis Myasthenia gravis Polymyositis

Correct Answer ( A ) Explanation: Giant cell arteritis (temporal arteritis) is a systemic autoimmune disorder. Pathologically, there is a granulomatous inflammation of large and medium-sized arteries. It is most common in women >50 years of age. Involvement may occur in any organ system, but the condition is characterized by subacute inflammation of the external carotid arterial system and vertebral arteries. Patients often describe a new temporal or diffuse headache that may be associated with transient visual loss, scalp tenderness, and jaw claudication. Patients may report a history of polymyalgia rheumatica (fever, myalgias, anorexia, weight loss, and arthralgias). If the diagnosis is suspected, treatment should be immediately initiated by administering steroids (prednisone 60 mg daily). Confirmation of the diagnosis is by temporal artery biopsy. Lab findings supportive of the diagnosis is an elevated ESR (50-100). Using the Westergren method, the value for a normal ESR is 30 mm/hr for a 60-year-old man; for women, top of normal range is age plus 10 divided by 2, so 35 mm/hr is the upper range of normal for a 60-year-old woman. Myasthenia gravis (C) is characterized by unilateral or bilateral ptosis, which is usually asymmetric (not amaurosis fugax), lid fasciculations, and lid retraction. Patients with myasthenia gravis frequently have eye muscle deviations with diplopia. Systemic manifestations include involvement of the jaw and neck, inability to hold the body erect because of weakness of the spinal muscles, and limb weakness. Multiple sclerosis (B) is associated with optic neuritis which is localized inflammation of the optic nerve sheath, resulting in reduced neuronal transmission and decreased visual acuity. Generally, there is a loss of color vision and red desaturation noticed by the patients. The symptoms generally worsen during the first few days and progressively improve over several weeks Polymyositis (D) is an inflammatory disease that has a bimodal distribution (age 10 to 15 and 45 to 60 years). The cause of inflammatory myopathies is unknown, but evidence suggests a genetic predisposition (associated with certain HLA markers) combined with an environmental insult, such as viruses, thereby initiating an autoimmune process. Patients usually experience progressive, symmetric, proximal muscle weakness with fatigue, malaise, and morning stiffness.

A previously healthy 30-year-old man presents to your clinic with questions about hepatitis C screening. He has no relevant medical history, no history of intravenous drug use, blood transfusions, high risk sexual behavior or needlestick injury. Which of the following is the most appropriate next step in management? Reassurance and counseling on prevention Referral to gastroenterology Screening with hepatitis C antibody test Screening with hepatitis C RNA test

Correct Answer ( A ) Explanation: Hepatitis C (HCV) infection can lead to cirrhosis of the liver and end-stage liver disease. Risk factor determination and screening for HCV are included as a part of overall preventive health maintenance. Risk factors for HCV infection include history of intravenous drug use or intranasal cocaine use, blood transfusion or receiving organ donation prior to July 1992, children of mothers who have HCV, needlestick injuries or exposure to HCV-positive blood, previous HIV infection, incarceration, sexual contact with a HCV-positive partner, and patients with evidence of liver disease. Patients determined to be at-risk for HCV should be screened. A one-time screening is also recommended for patients born between 1945 and 1965. Given the lack of risk factors in this patient, screening is not recommended and you should provide reassurance and counseling on prevention. Patients found to be positive for HCV should be referred to gastroenterology (B) for follow up. Patients at high-risk for HCV or those born between 1945 and 1965 should undergo initial screening with the HCV antibody test (C). A positive HCV antibody test should be followed by the HCV RNA test (D) to determine the genotype of the virus. This helps determine treatment options.

A woman complains of acute onset and rapidly progressive dark hair growth on her upper lip. She has also noticed a deepening of her voice, an increase in muscle bulk and a cessation of normal menses. Abdominal CT scanning reveals a large adrenal gland mass. Histopathologic evaluation shows the presence of malignant cells. An excess of which of the following class of hormones is most likely to be found in this patient? Androgens Catecholamines Eicosanoids Mineralocorticoids

Correct Answer ( A ) Explanation: Hirsutism, excessive terminal hair in androgen-dependent areas of a woman's body, affects up to 8% of women. It is usually a result of increased androgen levels. Underlying etiologies are many, including polycystic ovarian syndrome, late-onset adrenal hyperplasia and Cushing's syndrome. Adrenal neoplasia is also a cause, and a primary adrenal gland androgen-secreting neoplasia, typically a carcinoma of the androgen-producing cells, usually results in a rapid progression of hirsutism, development of male characteristics and cessation of menses. An excess of catecholamines (B), epinephrine and norepinephrine, can result from a pheochromocytoma. This adrenal medulla neoplasia causes hypertension, headache, palpitations, pallor and diaphoresis, not hirsutism and female masculinization. Eicosanoid hormones (C) include prostaglandins, leukotrienes and thromboxanes. These hormones are involved in the immunity and inflammation pathways, not the masculinization pathways. Mineralocorticoid (D) excess (hyperaldosteronism) causes hypertension, headaches and polydipsia.

A previously healthy 21-year-old man is found unresponsive by his roommates. On arrival to the Emergency Department, his pupils are pinpoint and oxygen saturation is 88% on room air. A blood gas reveals a pH of 7.25, PCO2 of 60 mm Hg, PO2 of 65 mm Hg, and a HCO3- of 26 mEq/L. Which of the following mechanisms is the most likely cause of his hypercapnia? Decreased respiratory drive Increased CO2 production Increased dead space ventilation Right-to-left shunt

Correct Answer ( A ) Explanation: Hypercapnia is defined as a PCO2 > 45 mm Hg. Hypercapnia is caused by alveolar hypoventilation. If ventilation is inadequate, CO2 is not exhaled at an appropriate rate and levels build up in the blood. The patient's elevated PCO2 and bicarbonate at the upper limit of normal indicates that he has an acute respiratory acidosis from hypoventilation. Hypoventilation results from a decreased respiratory rate, decreased tidal volume, or an increase in dead space ventilation. This can occur via numerous mechanisms: depressed central respiratory drive from a brainstem lesion, drug-induced depression of the respiratory center, neuromuscular disease (e.g. myasthenia gravis or Guillain-Barre syndrome), or conditions preventing adequate chest wall mobility (e.g obesity or tetanus). In this scenario of a young individual with pinpoint pupils, an opioid overdose causing respiratory depression is a likely cause of hypoventilation and resultant hypercapnia. Increased CO2 production (B) is incorrect. Hypercapnia is always caused by alveolar hypoventilation, not increased production. Increased dead space ventilation (C) is a cause of alveolar ventilation and can contribute to hypercapnia. However, in this clinical scenario opiate-induced respiratory depression is the more likely cause. Right-to-left shunt (D) causes hypoxemia, but not hypercapnia.

A six-year-old otherwise healthy male presents to the urgent care center for a lesion on his scalp. The barber noticed a mass on the back of his head when his mother took him for a haircut four days prior. She has been applying topical creams that have not led to any improvement. She does not know the name of the creams. His mother reports the mass has become more painful, and there is noticeable hair loss at the site. Physical examination of the scalp reveals a boggy 10 cm x 10 cm erythematous occipital swelling with multiple pustules and crusting. There is marked circular alopecia of this area, which is tender to palpation. There is also post-auricular lymphadenopathy. Which of the following would be the likely diagnosis? Alopecia areata Impetigo Kerion Seborrheic dermatitis

Correct Answer ( C ) Explanation: A kerion is an inflammatory type of tinea capitis. The host develops a T-cell-mediated hypersensitivity response to the dermatophyte causing the fungal infection. The causative organisms are mainly Microsporum and Trichophyton. It typically presents as an inflammatory scalp mass with pustules and hair loss and may have the appearance of an abscess. Diagnosis is made by clinical appearance and can be confirmed, if necessary, by potassium hydroxide wet mount of plucked hairs and scales. Cultures may also be obtained of the plucked hairs, if needed. Wood's lamp may be used and will be diagnostic of Microsporum if fluorescent green. Treatment consists of oral antifungals. Griseofulvin is considered first line of treatment and may be needed for as long as eight weeks. Patients should be followed closely by their physician to ensure proper response. Terbinafine, fluconazole, and itraconazole are considered second-line if griseofulvin fails. Topical antifungals and antifungal shampoos may be used to reduce transmission but should not be considered as treatment. The use of steroids is thought by some to reduce the inflammatory process and help prevent permanent scarring and alopecia but is still controversial. Alopecia areata (A) will also cause circular areas of hair loss but does not have pustules, scaling, or bogginess. There may be an erythematous border, which subsequently resolves. Exclamation mark hairs that taper from the end toward the scalp are pathognomonic. Alopecia areata also causes circumscribed areas of hair loss similar to tinea capitis, but alopecia areata does not cause scaling. Lesions may have an erythematous border in the early stages of the disease, but this reverses to normal color at later stages. The exclamation mark hairs seen in alopecia areata, in which broken hairs taper from the fractured end toward the skin surface, are pathognomonic. Impetigo (B) is a bacterial skin infection usually caused by Staphylococcus aureus or Streptococcus pyogenes and consists of groups of pustules with erythematous bases. It is more painful than a kerion and is not associated with hair loss. Seborrheic dermatitis (D) is a papulosquamous disorder seen on sebum-rich areas that may include the scalp, face, and trunk. It may have erythematous scaly lesions that may resemble tinea capitis, but it does not cause broken hairs or hair loss. There is also no associated bogginess.

A 32-year-old previously healthy woman presents to your office with complaints of headache, cough and episodes of fever, shaking, chills and sweating. She tells you that she was recently on a mission trip to Haiti and did not see a medical provider before her trip. Lab work is ordered and testing identifies Plasmodium falciparum on blood smear. Which of the following is the most appropriate therapy? Chloroquine Doxycycline Isoniazid Supportive care

Correct Answer ( A ) Explanation: Malaria is a mosquito-borne infection that is potentially fatal. The infection is caused by the Plasmodium protozoa and is transmitted by an infected female Anopheles mosquito. Cases of malaria in the United States are seen in patients who have traveled to malaria-endemic countries. Malaria is most prevalent in rural regions of countries with tropical climates. Patients planning a trip to a malaria-endemic region should be advised to take malaria prophylaxis to prevent infection. Clinical presentation includes several fever spikes per day, headache, cough, malaise, arthralgia, myalgia, chills and sweats. Diagnosis is made when the parasite is seen on blood smear with light microscopy. Regions where chloroquine-sensitive malaria is found include Haiti, the Dominican Republic, and certain parts of Central America. When infection with chloroquine-sensitive malaria is likely, treatment with the antimalarial agent chloroquine should be administered. Doxycycline (B) is a tetracycline antibiotic used in the treatment of Lyme disease and Rocky Mountain spotted fever, both tick-borne illnesses. Isoniazid (C) is used as part of the treatment regimen for tuberculosis. Supportive care (D) is used to treat yellow fever and dengue fever, which are both mosquito-borne illnesses. Patients with malaria should be treated with supportive care to help with symptoms, but should also undergo a treatment regimen based on country visited and potential for chloroquine resistance.

Which of the following correctly describes physiologic changes which occur in pregnancy? Blood volume increases Functional residual capacity increases Gastrointestinal motility increases Hemoglobin concentration increases

Correct Answer ( A ) Explanation: Pregnancy involves numerous physiologic changes in nearly every organ system to support the growth of the fetus. Cardiac output, blood volume and heart rate increase in pregnancy. Meanwhile, systemic vascular resistance decreases, resulting in a drop of 10 mm Hg in both systolic and diastolic blood pressure. Blood pressure reaches a nadir at about 28 weeks gestation and then slowly rises back to pre-pregnancy values. The hematologic systemic is also affected, with blood volume increasing by up to 30-40% in pregnancy. This increased blood volume not only supports that tremendous metabolic demands of the uteroplacental unit, but also protects the mother against the approximate 500 mL of blood loss that accompanies vaginal delivery. The increased blood volume leads to a dilutional anemia, as the rise in plasma volume is proportionately greater than the rise in red cell mass, and a hemoglobin level of 10.5 g/dL is considered normal. Leukocyte count is increased in pregnancy, and normal values range from 6000 to 16,000 cells/mm3 in the first and second trimesters. The pulmonary system is also affected. Tidal volume increases while functional residual capacity (FRC) is decreased as the gravid uterus displaces the diaphragm upwards. Physiologic hyperventilation leads to a mild respiratory alkalosis. Increased levels of progesterone and estrogen in pregnancy inhibit gastrointestinal motility, which can lead to constipation. Additionally, the lower esophageal sphincter tone is decreased, which predisposes pregnant patients to gastroesophageal reflux and aspiration. Functional residual capacity increases (B) is incorrect. FRC decreases in pregnancy because the gravid uterus displaces the diaphragm upwards. Gastrointestinal motility increases in pregnancy (C) is incorrect. Gastrointestinal motility is decreased in pregnancy. Hemoglobin concentration increases (D) in pregnancy is incorrect. Hemoglobin concentration is decreased as pregnancy is associated with a dilutional anemia caused by the increase in plasma volume which is proportionately greater than red cell volume.

A 19-year-old man with HIV presents with finger pain. Physical examination reveals the lesion shown above. What management is indicated? Acyclovir Cephalexin Incision and drainage Ritonovir

Correct Answer ( A ) Explanation: This patient has herpetic whitlow which is caused by the herpes virus. This is a self-limited condition. Immunocompetent individuals only require supportive care with analgesics and should cover the lesion to prevent autoinoculation. Immunocompromised individuals, as in the patient above, should be started on the antiviral, acyclovir. The lesion may be caused by HSV 1 or 2. Typically, the infection involves a single finger. It begins as pain or pruritus followed by the appearance of clear vesicles. These vesicles coalesce and can form an ulcer after 2 weeks. The diagnosis is made clinically but can be confirmed by a viral culture or Tzank smear from a scraping of an unroofed vesicle. The lesion resolves spontaneously in 3-4 weeks but recurrence occurs in 20% of patients. The main goals of treatment are to prevent oral inoculation or transmission and provide symptomatic relief. Cephalexin (B) is a first generation cephalosporin used in the treatment of bacterial skin and soft tissue infections. Incision and drainage (C) is contraindicated as it may lead to viral dissemination or bacterial superinfection. Ritonovir (D) is protease inhibitor used in the treatment of HIV infection. Although this patient is infected with HIV, administering ritonavir will not treat the herpetic whitlow.

A 15-year-old woman presents to the office with her mother concerned that she has not had a menstrual cycle. She is an avid runner, logging 20 miles per week. On exam she exhibits no breast development or axillary or genital hair. Her mother was 15 when she started her menstrual cycle. What is your next step? Begin work-up for primary amenorrhea Encourage her to stop exercising for 3 months Induce cycle with medroxyprogesterone (Provera®) No work up, but provide education on the topic

Correct Answer ( A ) Explanation: Primary amenorrhea is defined as failure of menarche by age 16 in a woman with apparently normal sexual development or by age 14 in a woman with no secondary sexual characteristics. Secondary amenorrhea is failure of menstruation after normal menses are established, with the caveat that at least 3 months have passed with apparently normal menses or 9 months have passed in a woman with oligomenorrhea. Primary amenorrhea can be caused by obstruction of the outflow tract, androgen insensitivity, gonadal dysgenesis, hyperprolactinemia, and dysfunction of the hypothalamus, pituitary, or thyroid. To evaluate a patient with primary amenorrhea, after a thorough clinical history, the physical examination must focus on development of secondary sexual characteristics (breast development, pubic, and axillary hair). Pregnancy as a cause of primary amenorrhea is less common than in secondary amenorrhea, although it should be excluded before any testing or imaging is initiated. Laboratory testing includes FSH, LH, TSH, and prolactin. If FSH is normal or reduced, this may mean the patient has chronic anovulation, functional hypothalamic amenorrhea, or polycystitc ovarian syndrome (PCOS). Increased FSH with breast development is likely secondary to ovarian failure. Increased FSH without secondary sexual characteristics may be caused by congenital agenesis of the ovaries. In the patient without a uterus, serum testosterone level and karyotype should be determined. In the presence of a uterus and normal secondary sexual characteristics, serum TSH levels should be evaluated. Inducing a cycle with medroxyprogesterone (Provera®) (C) is a strategy used for the treatment of secondary amenorrhea. Not initiating a work up (D) is inappropriate as primary amenorrhea is defined by age 14 if no secondary sexual characteristics have developed. Avid exercise (B) can be a cause of secondary amenorrhea but has no role in a patient diagnosed with primary amenorrhea.

A 29-year-old man comes to the clinic complaining of several months of worsening nasal obstruction and poor sense of smell. He has perennial allergic rhinitis, for which he takes diphenhydramine as needed. A nasal speculum exam shows a small, edematous, mucosa-covered mass in the left nasal passage. Which of the following is the best initial clinical intervention for this patient? Daily use of intranasal corticosteroids Daily use of non-sedating antihistamine Ethmoidectomy Polypectomy

Correct Answer ( A ) Explanation: Small nasal polyps can usually be managed initially for 1-3 months with daily intranasal corticosteroids to improve quality of life and reduce the need of an operation. Patients with significant impairment of nasal patency may also benefit from a short, tapered course of oral corticosteroids during this initial treatment phase. If patients fail to have improvement of symptoms after three months of intranasal steroids, surgery may be necessary. In healthy patients, a simple polypectomy is usually sufficient. However, recurrent cases may require a more complete procedure, such as an ethmoidectomy, for long-term relief. Nasal polyps present as edematous masses in the nasal passages that are covered by pale overlaying mucosa. Patients will complain of obstruction and may awaken with a dry mouth from their inability to breathe through their nose while sleeping. Sense of smell may be impaired. Risk factors for nasal polyps are varied. There is a connection between allergic rhinitis and nasal polyp formation. Allergen testing may be necessary to determine if this is contributing to polyp development. Patients with asthma and nasal polyps should use aspirin-containing products with caution as there is a known triad of patients with nasal polyps, asthma, and aspirin-sensitivity leading to potentially-severe bronchospasm (Samter's triad). Daily use of a non-sedating antihistamine (B) may be appropriate for managing this patient's allergic rhinitis; however, it will not lead to a decrease in size of the polyps or reduce need for surgery. An ethmoidectomy (C) is usually only necessary for patients with nasal polyps if they have had recurrent polyp formation or have complicating medical issues such as asthma. A polypectomy (D) is not the initial best step for this patient, as several months of intranasal corticosteroids may be able to reduce the need of having the polyp surgically removed.

A 17-year-old girl is seen in clinic due to vaginal discharge. She complains of yellow-green discharge and pruritus around her vaginal area. She is sexually active since last year and has had three partners. She denies taking any medication and uses condoms occasionally. Her last menstrual period was last week. On physical exam, you note frothy discharge with vaginal erythema and cervical hemorrhages. Which of the following is the next best step? Obtain a wet mount Obtain Herpes simplex virus PCR Obtain Rapid plasma Reagin (RPR) Provide reassurance and discharge home

Correct Answer ( A ) Explanation: The patient has signs and symptoms of acute cervicitis with findings most likely due to trichomoniasis. It presents with malodorous vaginal discharge, vulvovaginal irritation, dysuria, and dyspareunia. Physical exam may reveal a frothy discharge with vaginal erythema, and cervical hemorrhages (strawberry cervix) with the latter being a classic finding. The discharge usually has a pH of > 5. Trichomonas vaginalis may be recognized in vaginal secretions by using the wet mount technique which has a sensitivity of 60-70%. If motile trichomonads are not found, this should be sent for culture, which is the gold standard for diagnosis. Patients with trichomoniasis should be screened for other sexually transmitted infections such as chlamydia and gonorrhea. Metronidazole and tinidazole are used for treatment. Herpes simplex virus (HSV) PCR (B) would diagnose HSV which is characterized by multiple painful ulcers and vesicles on the labia, which this patient does not have. RPR (C) is a screening test for syphilis that can present with white-gray coalescing papules on labia known as condyloma lata or a painless chancre or both. The patient is symptomatic and requires treatment therefore reassurance (D) should not be all that is provided. The patient requires further diagnosis and treatment.

A 51-year-old woman presents with flank pain and fever. CT scan shows an obstructing 4 mm stone at the uretrovesicular junction. Urinalysis reveals 25-50 WBCs. What management is indicated? Antibiotics and inpatient urology consultation Antibiotics and outpatient follow up with urology Intravenous hydration and follow up with urology Pain control and follow up with urology

Correct Answer ( A ) Explanation: The patient presents with an obstructing stone and evidence of a urinary tract infection requiring inpatient management and urology consultation. Urolithiasis affects 5-15% of the world's population. The majority of stones are made of calcium oxalate with or without calcium phosphate. Ureteral obstruction from urolithiasis can cause impaired renal function and irreversible damage but this process takes time to develop. The most important cause of progressive renal damage is from associated infection. Patients with infected urolithiasis will typically present with renal colic type symptoms (colicky pain that is sharp, severe and intermittent with associated nausea and vomiting). Additionally, they will have fever and may demonstrate signs of sepsis (tachycardia, hypotension or shock). Patients with infected stones, particularly those that are obstructing, require hospital admission, intravenous antibiotics and urology consultation. In these patients, sepsis and kidney injury are possible risks and patients may require emergent drainage. Non-contrast CT scan of the abdomen and pelvis is the gold standard for diagnosis of urolithiasis. Antibiotics and follow up (B) is inadequate due to the risk of kidney damage and sepsis. Intravenous hydration (C) may be beneficial in patients with infection and or sepsis but is not adequate management in infected stones. Pain control with follow up (D) is appropriate for the treatment of urolithiasis without signs of infection.

A 34-year-old man presents with alcohol intoxication and left hand pain. The patient states that he fell asleep on the sidewalk in the snow. Examination reveals a swollen, erythematous left hand with clear blisters as seen above. The patient has decreased sensation and decreased range of motion. What therapy is indicated? Place hand in warm (37°C/98.6°F - 39°C/102.2°F) circulating water Place hand under hot (45°C/113°F - 52°C/125.6°F) running water Warm hand with circulating hot air Wrap hand in warm blankets

Correct Answer ( A ) Explanation: The patient presents with severe frostbite to the left hand from exposure and will require gentle, active rewarming of the extremity. During cold exposure, vasoconstriction occurs in an effort to conserve heat. As the temperature drops below 10°C, cutaneous sensation is compromised. With microvascular vasoconstriction, plasma begins to leak into the interstitial space. Ice crystals begin to form once the temperature approaches 0°C. Once crystals begin forming, intracellular osmolarity rises and cells begin to collapse and die. Blood flow begins to sludge followed by stasis and cessation of flow at the capillary level. Patients will often present with pain and decreased sensation (75%) but usually do not have frank frozen and insensate tissue. Frostbite, like burns, is classified into degrees of injury. First-degree frostbite is characterized by anesthesia and erythema. Second-degree frostbite will have superficial vesicles surrounded by edema. Third-degree frostbite produces hemorrhagic vesicles. Fourth-degree injuries extend deeper into osseous and muscle tissue. Optimal treatment should begin with removing all wet or cold clothing and assessing the patient for possible hypothermia. Any parts that are frozen should be submerged in warm circulating water (37°C - 39°C). Warming should not be initiated until it is certain that refreezing will not occur as this can cause more tissue damage. Passive rewarming with warm blankets (D) will not adequately increase the temperature to prevent further damage. Placing the frozen extremity in hot water (B) is often too painful for the patient to tolerate and may produce added heat injury. Application of dry heat (C) is also contraindicated as it leads to further damage and is poorly tolerated.

Which of the following is an appropriate treatment for the painful penile lesion pictured above? Acyclovir Azithromycin and ceftriaxone Benzathine penicillin G Doxycycline

Correct Answer ( A ) Explanation: The picture depicts the characteristic ulcerative lesions seen in genital herpes, the most common cause of sexually transmitted genital ulcerations in the United States. Lesions may be grouped, fluid-filled vesicles or ulcers on an erythematous base. Herpes simplex virus 2 (HSV-2) accounts for the majority of cases of genital herpes, but it may also be caused by herpes simplex virus 1 (HSV-1). Clinically, genital herpes manifests as either a primary infection or a local recurrence from reactivation of dormant virus in the spinal cord ganglia. Primary infection tends to be more severe with systemic symptoms, more copious genital lesions, and a prolonged clinical course. The diagnosis is largely clinical, although it can be confirmed with serologic testing, viral culture, or antigen retrieval from the lesions. The Tzanck test, once the mainstay of diagnosis, is no longer widely performed due to a lower sensitivity than newer testing has. Although outbreaks are self-limited and heal spontaneously, treatment with antiviral medication may decrease the duration of symptoms, decrease the amount of viral shedding (and thus infectivity), and reduce recurrences. It is also important to provide pain relief and educate the patient about sexually transmitted diseases. Azithromycin and ceftriaxone (B) is an appropriate treatment for chlamydia and chancroid. Benzathine penicillin (C) is an appropriate treatment for primary or secondary syphilis. Latent syphilis requires 2 additional treatments, each 1 week apart. Doxycycline (D) would be appropriate treatment for chlamydia and lymphogranuloma venereum.

A 23-year-old previously healthy man presents with abdominal cramping, nausea, vomiting and diarrhea 2 hours after eating at a picnic. Physical examination is unremarkable. What management is indicated? Anitiemetics and fluids Ciprofloxacin CT scan of the abdomen and pelvis Stool cultures

Correct Answer ( A ) Explanation: This patient presents with a rapid onset gastroenteritis that is most consistent with Staphylococcal food poisoning resulting from ingestion of preformed enterotoxin. Staphylococcus-related food poisoning occurs when the bacteria multiplies in food prior to ingestion and creates enterotoxin. Protein rich foods (particularly ham, eggs, mayonnaise containing salads) support the bacteria's growth. Patients experience a sudden onset of nausea, vomiting and diarrhea that begins 1 to 6 hours after ingestion of the contaminated food. The rapid onset is due to the fact that the toxin has already been formed prior to ingestion. Patients often recover quickly; usually between 6 and 8 hours of symptom onset and frequently do not present for medical treatment. Often, there will be outbreaks of Staphylococcal-related food poisoning from multiple people ingesting the same contaminated food. Treatment focuses on rehydration and treatment of nausea and vomiting if it is still present on presentation to the Emergency Department. Antibiotics (B) are not beneficial in this type of gastroenteritis as it is not caused by viable organisms. CT scan (C) does not add to diagnosis or guide treatment. Stool cultures (D) will also not be beneficial as there is no live bacteria to culture.

A 68-year-old man presents to the emergency department with left lower quadrant pain and fever for one day duration. Laboratory results reveal a WBC 14,800, Hb 12.0 g/dL, sodium 138 mEq/L, potassium 4.0 mEq/L, and creatinine 1.0 mg/dL Which of the following studies is contraindicated in the workup of this patient? Colonoscopy Contrast computed tomography Magnetic resonance imaging Ultrasound

Correct Answer ( A ) Explanation: This patient's presentation is consistent with acute diverticulitis. Colonoscopy is contraindicated in acute diverticulitis due to the risk of bowel perforation. Acute diverticulitis typically presents with constipation or diarrhea along with LLQ tenderness. Age and low-fiber diet are the most important risk factors. CT is the preferred diagnostic imaging in suspected acute diverticulitis. Computed tomography (B) is the preferred test in evaluating clinically suspected diverticulitis. CT evaluates the severity and extent of disease and may also diagnose other causes of LLQ pain that can mimic acute diverticulitis. Although MRI (C) is not the preferred modality for the diagnosis of acute diverticulitis, colonoscopy is more clearly contraindicated due to the risk of perforation. Ultrasound (D) may be used in the workup of this patient, especially if his creatinine began to rise even higher.

A patient presents with weight loss, nervousness, and palpitations. During physical examination, which of the following signs suggests hyperthyroidism? Anterior neck bruit Coarse, dry, scaling skin Distal muscle weakness Periorbital erythema

Correct Answer ( A ) Explanation: Thyroid gland examination involves inspection, palpation and auscultation. The exam begins with extension of the neck so the anterior surface can be inspected for asymmetry. Asking the patient to swallow may enhance visualization of thyroid enlargement. Palpation follows, assessing for nodules, tenderness, asymmetry, size and consistency. Two approaches can be used, facing the patient or behind the patient. It is important during palpation to have the sternocleidomastoid muscles relaxed. The thyroid lobes should be small, firm, smooth, and nodule-free. A coarse, gritty sensation suggests inflammation. If nodules are present, the clinician should note number, consistency and smoothness. If the gland is enlarged, consider thyroiditis or hyperthyroidism. Auscultation of an enlarged gland (goiter) may reveal a bruit or soft rushing sound if the patient is in a hypermetabolic state, indicating increased blood flow. Goiter and gland tenderness suggests a hyperthyroid state. Coarse, dry, scaling skin (B) is more commonly seen in hypothyroidism. Distal muscle weakness (C) is common in peripheral neuropathies. Hyperthyroidism is associated with proximal, not distal, muscle weakness. This may be appreciated as shoulder or hip girdle weakness in the setting of normal strength hands and ankles. Periorbital erythema (D) is seen with periorbital cellulitis not hyperthyroidism.

A 38-year-old man presents to the ED with his friend. They were playing racquetball and the man was hit directly in the eye with the ball. Examination reveals limitation of upward gaze, periorbital erythema, and subconjunctival hemorrhage of right eye. Imaging of the orbit would most likely show a fracture of which bone? Frontal Maxillary Sphenoid Zygomatic

Correct Answer ( B ) Explanation: A blowout fracture typically refers to a fracture of the inferior orbital wall fracture. The most common causes are direct blunt injury such as a fist, sports' ball injury, and motor vehicle collisions. Symptoms include upward gaze diplopia, decreased extraoccular eye movements due to entrapment of the inferior rectus muscle, periorbital ecchymoses, eyelid edema and subconjunctival hemorrhage. Suspicion is based on the history, while diagnosis requires radiographic or computed tomography confirmation. Typical radiographic findings include a teardrop sign (a mass of herniated orbital contents, fat and the inferior rectus muscle), orbital lucency, and sinus air-fluid levels. Pure blowout fractures occur most frequently in the inferior orbital floor. Since the floor (inferior wall) is the most common site of orbital blowout, the maxillary bone is most commonly fractured. The medial wall is the second most commonly fractured area. The orbit is a pyramidal-like structure made up of 6 areas: superior orbital wall (frontal bone) (A), inferior orbital wall or floor (maxillary bone), medial wall (lacrimal and ethmoid bones), lateral wall (zygomatic bone) (D), apex (sphenoid bone) (C), and base (anterior eyeball surface).

A 28-year-old south Asian immigrant who is in her second trimester of her first pregnancy presents to the emergency department complaining of worsening dyspnea, orthopnea and lower extremity edema. She has never experienced anything like this before. She has no past medical history; however, she admits to frequent sore throats and ear infections as a child. Which of the following is most likely to be heard on auscultatory exam? A diastolic decrescendo murmur heard at the left lower sternal border A diastolic low-pitched decrescendo murmur best heard at the cardiac apex A holosystolic murmur heard best at the cardiac apex A systolic crescendo-decrescendo murmur best heard at the right upper sternal border

Correct Answer ( B ) Explanation: A diastolic low-pitched decrescendo murmur best heard at the cardiac apex would be the most likely auscultatory finding on exam. This woman likely has mitral stenosis secondary to rheumatic heart disease. Mitral stenosis encountered in women of childbearing age is nearly always rheumatic in origin. Maternal and perinatal complications during pregnancy in women with mitral stenosis reflect the unfavorable interaction between the normal cardiovascular changes of pregnancy and the stenotic mitral valve. Pregnancy is a high flow state and blood volume, cardiac output, and heart rate are increased. In pregnant patients with mitral stenosis, this increases the pressure across the mitral valve and can lead to pulmonary edema and other signs and symptoms of heart failure. Although mitral stenosis is relatively uncommon in developed countries, it is a common condition in pregnant women with heart disease in areas where rheumatic heart disease is prevalent. Although rheumatic heart disease may affect other valves, including aortic and tricuspid, the mitral valve is the predominant lesion and repeated attacks of rheumatic fever over time causes mitral valve stenosis. A systolic crescendo-decrescendo murmur best heard at the right upper sternal border (D) is characteristic of aortic stenosis. A diastolic decrescendo murmur heard at the left lower sternal border (A) classically describes aortic regurgitation. A holosystolic murmur heard best at the cardiac apex (C) is associated with mitral regurgitation.

A 19-year-old sexually active woman comes to your office for a routine checkup. She is generally healthy with no chronic conditions and does not smoke. Which of the following conditions should you perform a screening test for? Cervical cancer Chlamydia infection Human Papillomavirus (HPV) Hypercholesterolemia

Correct Answer ( B ) Explanation: According to the U.S. Preventive Services Task Force (USPSTF), there is good evidence that screening for Chlamydia infection in women who are at increased risk can reduce the incidence of pelvic inflammatory disease, while the harms are minimal. The evidence regarding screening under age 21 for cervical cancer (A) with Pap testing or human papillomavirus (HPV) (C) testing, however, shows that the harms outweigh any possible benefits. Harms include over diagnosis and over treatment, including invasive cervical procedures that can affect future pregnancy outcomes. In addition, there is adequate evidence that screening women younger than 21 years of age (regardless of sexual history) does not reduce the incidence of cervical cancer or mortality compared with beginning screening at age 21. The USPSTF concludes that the evidence is insufficient to recommend for or against routine screening for lipid disorders (D).

You suspect a new patient has systemic lupus erythematous (SLE) based on symptomatology and physical exam. In addition to an ANA, what laboratory tests should be ordered to help confirm the diagnosis of SLE? Anti-Ro/SSA and anti-La/SSB Anti-smith antibody and anti-double-stranded DNA Rheumatoid factor and anti-CCP Sedimentation rate and CRP

Correct Answer ( B ) Explanation: Antibodies to an RNA-protein complex called Sm (anti-Smith antibody) and to double-stranded DNA (dsDNA) are highly associated with systemic lupus erythematous (SLE), both being 95% specific. ANA, anti-double-stranded DNA, and antiphospholipid antibody are all markers for SLE, but these test results may be misleading if not considered in clinical context. Between 2% - 5% of patients with SLE are ANA negative, whereas 5% of the normal population, and up to 20% of healthy young women are ANA positive. SLE is most common in women of reproductive age (15-40 years). The female/male ratio is approximately 2:1 before puberty and 4:1 after puberty. However, SLE is seen in all ages, including infants and older adults. In these two subpopulations, the female/male ratio is only 2:1. SLE affects approximately 1 in 1000-2500 in the general population, but disease incidence in African American and Latino women is much higher (up to 1 in 250 in African American women ages 18-65 years). The 5-year survival after diagnosis is 90%. Secondary to the wide variety of presentations, the American College of Rheumatology created a classification system to standardize the diagnosis of SLE. To confirm a diagnosis of SLE, patients must have at least 4 of the 11 criteria present, either serially or simultaneously. Constitutional symptoms found in patients with SLE include fatigue, malaise, fever, and weight loss. Thus, the physician has a critical role to play in the diagnosis of this multisystem disease, which must be differentiated from disease processes such as: HIV (often is false positive in SLE), subacute bacterial endocarditis, and other connective tissue diseases including vasculitis, rheumatoid arthritis, mixed connective tissue disease, and malignancies such as lymphomas. SLE is characterized by specific organ system abnormalities. The disease most often affects the skin, joints, kidneys, CNS, GI tract, and lungs, with a varying spectrum of disease severity and unpredictable clinical course. SLE is truly systemic. Other asymptomatic organ involvement must be investigated during active periods of disease, regardless of the presenting sign or symptom. Anti-Ro SSA and anti-la/SSB (A) are commonly used to help diagnose Sjogrens syndrome not lupus. Rheumatoid factor and anti-CCP (C) will be seen in patients with rheumatoid arthritis and not lupus. Sedimentation rate and CRP (D) are non-specific markers of inflammation and would not be helpful in pinpointing lupus as these can be elevated in various connective tissue diseases.

A 53-year-old man with a history of atrial fibrillation and hypertension presents with severe abdominal pain. He states that the pain has been there for 3 days but got more severe today. Over the last 3 days, he has been unable to eat because the pain occurs after eating. Vitals signs are T 99.7F, HR 123, BP 101/66, RR 24. Examination reveals an uncomfortable patient with diffuse mild abdominal tenderness to palpation without rebound or guarding. Stool guaiac is positive and the serum lactate is 4.8 mg/dL. A surgical consultation is requested. Which of the following represents the appropriate management? Obtain abdominal X-rays Obtain CT angiogram of the abdomen and pelvis Obtain CT scan of the abdomen and pelvis without IV contrast Obtain right upper quadrant abdominal ultrasound

Correct Answer ( B ) Explanation: This patient's presentation is highly suggestive of mesenteric ischemia, which is best diagnosed by CT angiogram. Once ischemia has progressed to infarction, mortality climbs to 70% underscoring the need for rapid diagnosis and management. Mesenteric arterial embolism is the most common cause of mesenteric ischemia. Risk factors for arterial embolus include coronary artery disease, valvular heart disease and arrhythmias - particularly atrial fibrillation. The classic presentation of mesenteric ischemia is that of an elderly patient who presents with intermittent abdominal pain that is increased with eating. The pain is out of proportion to examination (meaning that the patient complains of severe pain but the exam reveals mild to moderate tenderness and may lack peritoneal signs). Additionally, patients will often have guaiac positive stool testing especially as the bowel becomes more ischemic. An elevated lactate level is also highly suggestive of mesenteric ischemia and has a high sensitivity. Because of the high morbidity and mortality associated with this disease, early surgical consultation and definitive imaging is central to management. The study of choice is either conventional angiography or CT angiography (more easily accessible). Management focuses on aggressive resuscitation, supportive care and early diagnosis. Abdominal radiographs (A) are useful in eliminating other possible diagnoses including small bowel obstruction and bowel perforation but they are not sensitive in diagnosing either of these diseases or mesenteric ischemia. Angiography or CT angiogram should not be delayed while obtaining plain radiographs. Traditional CT scan of the abdomen and pelvis either with or without contrast (C) may show signs of ischemia like bowel wall edema or intramural gas but will not show where the vascular obstruction lies and is not sensitive for detecting mesenteric ischemia. A right upper quadrant ultrasound (D) is the diagnostic modality of choice for biliary disease (cholelithiasis, cholecystitis etc.) but has not been shown to be useful in diagnosing mesenteric ischemia.

A 16-year-old girl presents to the ED for a minor laceration repair of her forehead after a picture frame fell off the wall and hit her. Her vital signs are blood pressure 175/75 mm Hg, HR 80, and RR 14. The patient states on review of systems that she has had headaches, chest pain, and fatigue over the previous few months. You note a systolic murmur in the left infraclavicular area and under the left scapula. Which of the following is the most important next step in management? Arrange for outpatient follow-up for repeat blood pressure Obtain blood pressure readings in the upper and lower extremities Order a renal ultrasound to evaluate for fibromuscular dysplasia of the renal arteries Send electrolytes and urinalysis in preparation to begin antihypertensive therapy

Correct Answer ( B ) Explanation: Coarctation of the aorta usually presents as congestive heart failure and cardiogenic shock in the neonatal period due to increased afterload caused by narrowing of the aorta. In mild cases, patients can develop arterial collateral vessels that partially bypass the aortic obstruction. These patients usually remain asymptomatic. The diagnosis is often made when incidental hypertension is noted during the evaluation of other problems such as trauma or routine illness. In older children, adolescents, and adults, coarctation of the aorta is best diagnosed clinically by simultaneous palpation of femoral and brachial pulses. Blood pressure in both arms and one leg must be determined; a pressure difference of more than 20 mm Hg in favor of the arms may be considered evidence of coarctation of the aorta. The chest radiograph in these patients may demonstrate rib notching secondary to collateral vessels. Referring the patient for follow-up (A) is always a good idea but, in this case, would delay her diagnosis of coarctation of the aorta. Many patients exhibit "white coat hypertension," and a repeat blood pressure is usually normal. However, hypertension in the pediatric age group may have a secondary cause and mandates a thorough evaluation. Renal artery stenosis due to fibromuscular dysplasia (C) is a secondary cause of hypertension seen most commonly in young female patients. Renal ultrasound may show stenosis of the renal artery. However, patients with this condition will have no pulse or blood pressure discrepancies. Patients with hypertension should be started on antihypertensive medications (D) to reduce the future risk of cardiovascular disease. While this patient may require such therapy in the future, her underlying pathology (i.e., coarctation of the aorta) should first be addressed.

Over the last 7 months a developmentally normal 13-year-old girl has had intermittent abdominal pain, which has made her quite irritable. The abdominal pain is associated with arthralgias and general malaise. Review of systems reveals that she has lost 5 kg (11 lb) and has painful bowel movements. Which one of the following is the most likely cause of these symptoms? Celiac disease (gluten enteropathy) Crohn's disease Irritable bowel syndrome Ulcerative colitis

Correct Answer ( B ) Explanation: Crohn's disease is the most common chronic inflammatory bowel disease which occurs during adolescence and young adulthood, with a second peak at 50-80 years of age. The manifestations of Crohn's disease are dependent on the site of involvement, but systemic signs and symptoms are more common than with ulcerative colitis. Crohn's disease presents with chronic diarrhea, crampy abdominal pain, fever, weight loss, and fatigue. Perianal disease (anal fissures, perirectal abscesses, and anorectal fistulas) is also common in Crohn's disease. Crohn's disease may affect any part of the GI tract, from the mouth to the anus, but typically affects the small bowel and colon. Irritable colon and celiac disease may mimic symptoms of Crohn's disease, but objective findings of weight loss and anal lesions are extremely uncommon. Irritable bowel syndrome (C) is characterized by abdominal discomfort or pain associated with altered bowel habits for at least three days per month in the previous three months, with the absence of organic disease. Celiac sprue (A) is also known as celiac disease or gluten-sensitive enteropathy and is a chronic disorder of the digestive tract that results in an inability to tolerate gliadin, the alcohol-soluble fraction of gluten. Ulcerative colitis (D) is one of the two major types of inflammatory bowel disease along with Crohn's disease. Unlike Crohn's disease, which can affect any part of the gastrointestinal tract, ulcerative colitis characteristically involves only the large bowel.

What is the most common chronic illness of childhood? Asthma Dental caries Diabetes mellitus Obesity

Correct Answer ( B ) Explanation: Dental caries are the result of decaying tooth enamel. Bacteria on the teeth break down foods, producing acids that destroy the tooth enamel and result in dental caries. Dental caries are preventable but are still the most common chronic illness of childhood. Dental screening by the pediatric primary care provider should occur when the teeth first erupt, so that children at greater risk for development of dental disease may be referred for early dental evaluation. Ideally, initiation of dental care should occur as soon as possible following the child's first birthday. If there are no available pediatric dental providers accepting children younger than the age of three years, the pediatric primary care provider may continue to provide counseling and preventive dental care until the child is able to establish dental care. Asthma (A) is a chronic, inflammatory lung condition and is among the most common chronic illnesses of childhood, however dental caries are more common. The rate of diabetes mellitus (C) type 2 in children is increasing and is linked to a rise in childhood obesity (D). While diabetes mellitus and childhood obesity are increasing public health concerns, they are both less common than dental caries.

What structure helps differentiate between internal and external hemorrhoids? Anal verge Dentate line Linea alba Perineum

Correct Answer ( B ) Explanation: Hemorrhoids are submucosal vascular beds located in the anal and rectal canal that assist with defecation and the sensation of anorectal fullness. These vascular beds can occur in insensate areas above the dentate line as internal hemorrhoids or below the dentate line as external hemorrhoids in exquisitely sensitive areas. Hemorrhoid development may result from genetic factors, aging or serial local trauma. One study found that 4.4% of the U.S. population, or 10 million people, complain of hemorrhoid disease. Painless, bright-red rectal bleeding is the primary symptom of internal hemorrhoids. Anoscopy can be used for identification of the site of bleeding acutely. Dietary management consisting of adequate fluid and fiber intake is the primary noninvasive treatment of symptomatic hemorrhoids. Hemorrhoids are classified according to the dentate line based on where the hemorrhoid appears to start not at the anal verge (A). The linea alba (C) is a median vertical tendinous line on the abdomen, formed by fibers from the aponeuroses of the two rectus abdominis muscles and extending from the xiphoid process to the pubic symphysis, having nothing to do with hemorrhoids. The perineum (D) is the skin between the scrotum and anus and is not the distinguishing feature for hemorrhoids. One Step Further

A 32-year-old woman presents to your office for her annual exam. She reveals that one month ago her 6-year-old daughter was killed in a motor vehicle accident. Since the accident she has been crying uncontrollably, can't sleep through the night, and sometimes sees her daughter walking in the upstairs hall. Which of the following is a risk factor for the development of poor bereavement outcomes? Crying uncontrollably Death of a child Insomnia Occurrence of visual hallucinations

Correct Answer ( B ) Explanation: Prolonged or complicated grief is characterized by persistent, disruptive emotional responses for at least six months after the death of a loved one. Risk factors include death of a child, past history of mental health issues, dependent relationship with the deceased, and poor social supports. Symptoms include difficulty with moving on and accepting the death, bitterness, numbness or detachment, agitation or being on edge, lack of trust, feelings of emptiness, and a sense that the future holds no meaning. Prolonged or complicated grief can lead to bereavement-related depression or medical sequelae such as exacerbation of chronic disease and substance abuse. Complicated grief can be difficult to differentiate from depression, therefore patients meeting criteria for complicated grief should be referred to a psychiatrist for evaluation. Providers should also encourage individuals with complicated grief to practice self-care, develop new routines and relationships, and provide referrals to support groups. Crying uncontrollably (A), insomnia (C) and visual hallucinations (D) are all normal grief reactions. Reactions to grief come in waves, with individuals feeling good one day and extremely sad the next. The distressing emotions of normal grief slowly decrease in intensity, with the individual eventually coming to accept the loss. Impairments caused by grief reactions begin to resolve by six months, although important events such as birthdays and anniversaries may cause grief reactions to return for years after the death.

Thyroid storm is a severe variant of thyrotoxicosis and is best diagnosed by which of the following? Antithyroid antibodies Clinical findings Levels of circulating T4 Levels of TSH

Correct Answer ( B ) Explanation: The diagnosis of thyroid storm is based on clinical findings. Thyroid storm is often precipitated by infection, which can cause symptoms that mask a thyrotoxic state. Clinical findings in thyroid storm include hyperpyrexia (>38.8°C); tachycardia out of proportion to temperature; gastrointestinal (GI) dysfunction (nausea, vomiting, diarrhea, jaundice); and CNS dysfunction (marked hyperirritability, anxiety, confusion, apathy, coma). There is usually pronounced decompensation of 1 or more organ systems. Any patient presenting with goiter, fever, and marked tachycardia should be considered to be in thyroid storm and treated accordingly. Admission to the intensive care unit and consultation with an endocrinologist is appropriate. Treatment of thyroid storm includes beta-blockers, antithyroid drugs (propylthiouracil), glucocorticoids, iodide, antipyretics, aggressive fluid replacement, and identification and treatment of any precipitating process. Lugol's solution or potassium iodide inhibits release of T4 into the peripheral circulation. If either is used, it should be given only after loading doses of antithyroid drugs to block iodine-induced synthesis of T4. Lithium also has an antithyroid effect and can be used in severe cases of thyroid storm. Severe thyrotoxic symptoms, unresponsive to all these regimens, may respond to sodium ipodate at 500 mg/day. Thyroid storm is a clinical diagnosis. Although antithyroid antibodies (A) and the levels of T4 (C) and TSH (D) can help to identify a hyperthyroid state, thyroid storm is a clinical diagnosis. Any delay in diagnosis due to waiting for laboratory results to return can be life-threatening.

A 47-year-old woman with diabetes mellitus is found to have latent tuberculosis during a routine employee health purified protein derivative (PPD) test. She should receive supplementation with pyridoxine (vitamin B6) if her drug regimen contains which of the following medications? Ethambutol Isoniazid Pyrazinamide Rifampin

Correct Answer ( B ) Explanation: The use of isoniazid requires supplementation with pyridoxine in high risk patients (eg. those with diabetes, HIV, chronic alcoholism, renal disease or pregnant or breast-feeding women) to prevent the development of peripheral neuropathies during treatment of active or latent tuberculosis. It is crucial that latent tuberculosis be treated to control and, ideally, eliminate tuberculosis in the United States. A tuberculin skin test, or Mantoux test, in which a purified protein derivative (PPD) is injected under the skin, is most widely used to determine previous exposure to tuberculosis. Several at-risk groups should receive this test, such as HIV-positive patients, recent immigrants from countries with a high tuberculosis prevalence, and residents or workers in prisons, healthcare facilities, homeless shelters, and other high-congregate settings. If a patient's PPD demonstrates induration sufficient to diagnose tuberculosis, a follow-up analysis of sputum should be used to definitively diagnose the condition. In HIV-negative patient, pulmonary tuberculosis can usually be eradicated with a 6-month anti-tuberculosis drug regimen. The first two months should consist of isoniazid, pyrazinamide, ethambutol, and rifampin. If the M. tuberculosis isolate is found to be sensitive to isoniazid and rifampin, the pyrazinamide and ethambutol may be stopped for the final 4 months of therapy. In pregnant patients, pyrazinamide should not be used during the initial 2 months, and the isoniazid/rifampin regimen should continue for an additional 3 months. A longer course of therapy is generally recommended for HIV-positive patients as well. As a rule, patients whose sputum cultures do not become negative within 3 months should be evaluated for drug-resistant organisms or nonadherence to therapy. Ethambutol (A), a bacteriostatic anti-tuberculosis drug, is more likely to cause optic neuritis. This can be distinguished with a visual acuity screening and red-green discrimination test, and is usually reversible with discontinuation of the drug. Pyrazinamide (C), a bactericidal anti-tuberculosis drug, is known to cause hyperuricemia, hepatotoxicity, and joint pain. Side effects should be monitored during treatment with uric acid and liver function tests. Rifampin (D), a bactericidal anti-tuberculosis drug, may also cause hepatitis, as well as flu-like symptoms and poor clotting. Monitoring parameters should include liver function tests and complete blood counts. Patients should also be cautioned that this drug can color bodily secretions orange and stain contact lenses.

A 65-year-old man is brought to the ED after a fall. He says he has had trouble walking "for a while." His examination is significant for normal cranial nerve function, normal strength, a resting tremor, and difficulty stopping when he is walking. What is the cellular pathology associated with this condition? Demyelination Lewy bodies Loss of anterior horn cells Neurofibrillary tangles

Correct Answer ( B ) Explanation: This patient is showing signs of Parkinson's disease, a chronic neurologic condition. It is characterized by intracellular cytoplasmic inclusions called Lewy bodies, dopaminergic neuron loss in the substantia nigra, and depigmentation along with gliosis of pigmented areas in the midbrain. The symptoms can be remembered with the mnemonic TRAP: resting Tremor, cogwheel Rigidity, Akinesia, and impairment of Posture or equilibrium. It is treated with drugs to increase central dopamine levels, dopamine receptor agonists (carbidopa and levodopa), and anticholinergic agents (which offset the movement effects induced by diminished dopamine). There is no cure for Parkinson's disease. The most common cause of death is respiratory failure. Demyelination (A) is associated with multiple sclerosis and Guillain-Barré syndrome. Loss of anterior horn cells (C) is associated with amyotrophic lateral sclerosis (ALS), a progressive degenerative of upper and lower motor neuron disease. Neurofibrillary tangles (D) and amyloid plaques are thought to be responsible for Alzheimer's disease, which is a progressive cognitive disorder.

A 25-year-old man is found to have a tibia fracture after he was struck by a motor vehicle while crossing the road. An X-ray of his leg is seen above. What is the orientation of this fracture? Comminuted Greenstick Segmental Spiral

Correct Answer ( C ) Explanation: Characterizing tibia shaft fractures is most important when communicating with an orthopedic consultant. A single, large, free-floating segment of tibia bone between two well-defined fracture lines is a segmental fracture. In addition to orientation, fractures should also be described in terms of whether the fracture is open or closed, location of the fracture in the shaft, displacement and separation, and angulation. The fracture pattern will give a clue to the force that caused the injury. The tibia shaft requires a high degree of force to fracture and should prompt investigation for additional injuries. Tibia fractures are often open injuries because of the minimal amount of subcutaneous tissue between the fracture and the skin. Open fractures require immediate orthopedic consult and administration of IV antibiotics along with appropriate analgesia and splinting. Segmental fractures are commonly mischaracterized as comminuted (A), which is actually a splintering or shattering of the bone from a high-energy mechanism of injury. Greenstick (B) and torus fractures occur exclusively in children. Spiral (D) fractures are the result of rotational forces. The fracture line spirals along the shaft of the long bone.

A 3-year-old girl presents to the ED with her mom and grandmother for evaluation of vaginal spotting. The child has no other complaints and specifically denies sexual abuse or trauma when questioned alone. After watching a news special on sexual abuse, mom is concerned that her daughter may have been sexually assaulted, despite no specific concern. The child is acting appropriately and there are no external lesions or signs of trauma. Internal pelvic exam is difficult due to the patient's age. Which of the following statements is true? Obtain a pelvic x-ray prior to attempting a physical exam She is the victim of abuse; do no further evaluation without a trained nurse present and contacting authorities She may have a vaginal foreign body; consider a nasal speculum to attempt visualization and removal Vaginitis is an unusual diagnosis in this age group

Correct Answer ( C ) Explanation: Children often fear parental disapproval of a vaginal foreign body placement. This often leads to a delay in diagnosis until secondary signs (such as vaginal bleeding, foul smelling discharge, or purulent drainage) are noted by parents. The physical exam is often quite difficult owing to patient anxiety, small anatomic size, and parental concerns about sexual or physical abuse. However, a thorough vaginal exam is indicated in patients with vaginal bleeding or discharge. Use of a nasal speculum, procedural sedation, or trained assistant (such as a child life specialist) may facilitate this exam. Although plain films (A) may identify certain radiopaque objects, they are often of limited diagnostic value. A negative film does not disprove the presence of a radiolucent vaginal foreign body. Sexual and physical abuse (B) is always an important consideration when evaluating children with genitourinary or rectal complaints. When in doubt, ethical and legal requirements obligate the notification of appropriate child protection services. However, in this case, there is no evidence of abuse. Foreign body or vaginitis is a much more likely diagnosis. Vulvovaginitis (D) is a very common diagnosis in this age range owing to a lack of estrogen, developing toilet hygiene, and frequent use of baths, perfumed soaps, and bubbles.

A mother brings her 3-year-old previously healthy son to the ED with a complaint of pinkeye. This began acutely 2 days ago. He has never had these symptoms before. Examination reveals scleral erythema and purulent discharge. Which of the following antibiotics do you recommend? Doxycycline Neomycin Ofloxacin Tetracycline

Correct Answer ( C ) Explanation: Conjunctivitis, or pinkeye, is a common reason that patients seek medical attention. This mucus membrane inflammation occurs within the coverings of the surface of the sclera and conjunctivae and the inner surface of the eyelids (palpebral conjunctiva). There is a near equal frequency of bacterial and viral etiologies, however other causes include fungal, parasitic, chemical, chlamydial and allergic sources. The major symptoms are erythema (redness, injection), discharge, irritation (pruritis and foreign body sensation) and possibly photophobia. Most cases are benign and self-limited. However, antimicrobial therapy is typically recommended due to the difficulty in distinguishing between viral and bacterial causes. Antibiotic solutions contain sulfacetamide, erythromycin, gentamicin, ciprofloxacin or ofloxacin. Neomycin (B) solutions should be avoided as there is a high incidence of hypersensitivity reactions. Conjunctivitis due to Chlamydia trachomatis should be suspected in remission-recurrent (i.e., chronic relapsing) cases, or in persons with sexually transmitted infections. Neither criteria exist in the above patient. Suspected chlamydial conjunctivitis is treated with tetracycline (D), doxycycline (A), azithromycin or erythromycin.

An 8-year-old boy with a history of cystic fibrosis presents with increased cough and fever. He has been doing well and has not been hospitalized for pneumonia in the last seven months. His X-ray demonstrates pneumonia and he requires supplemental oxygen. Which of the following antibiotic regimens is most appropriate? Ceftriaxone and azithromycin Moxifloxacin Piperacillin/tazobactam Vancomycin and ceftriaxone

Correct Answer ( C ) Explanation: Cystic fibrosis (CF) is caused by a mutation in the CF transmembrane conductance regulator gene and transmitted in an autosomal recessive pattern. The mutation leads to impaired chloride transport across the airway epithelium causing multiple problems within the lung including impaired ciliary clearance of mucus, decreased antimicrobial effect of the airway surface, increased bacterial adherence, and impaired secretion of cytokines. As a result, patients develop recurrent bacterial pneumonias over the course of their life. Approximately 80% of patients with cystic fibrosis are colonized with Pseudomonas aeruginosa by age 18. Therefore, active pneumonia infections are treated with broad-spectrum antibiotics with adequate activity against Pseudomonas, such as Piperacillin/tazobactam. It is also important to evaluate previous culture data for resistance patterns as patients with cystic fibrosis are on frequent antibiotics including some prophylactic agents leading to growing resistance as they age. In addition to antibiotics, patients are treated with bronchodilators and mucolytics to improve airway clearance and chest physiotherapy. Ceftriaxone and azithromycin (A) are appropriate antibiotics for the treatment of community-acquired pneumonia. However, none of these medications provide coverage against Pseudomonas. Moxifloxacin (B) is a fluroquinolone which is contraindicated in children due to disruption of tendon growth. Vancomycin and ceftriaxone (D) are not commonly administered together for the treatment of pneumonia. Vancomycin is an appropriate antibiotic to consider for coverage of methicillin-resistant Staphylococcus aureus and patients with CF are at risk for this infection because of repeated antibiotic use. Ceftriaxone is a third-generation cephalosporin used for community-acquired pneumonia but does not provide the essential Pseudomonal coverage this patient needs.

A 15-year-old boy who runs on his high school cross-country team presents to your office complaining of pain just below his right knee. He states that the pain started 3 months ago and is worse at night. He denies any recent injuries or trauma. You order an X-ray and obtain the image below. Which of the following is the most likely diagnosis? Ewing sarcoma Osgood-Schlatter disease Osteosarcoma Patellar tendonitis

Correct Answer ( C ) Explanation: Osteosarcoma is the third most common malignancy of adolescents and is the most common malignant bone tumor in adolescents. It often occurs around the time of growth spurts (adolescents) and patients usually present after several months of pain. Teenagers who are active in sports tend to complain about pain in their lower femur, or right below the knee. If the tumor is large, it can appear as a swelling. The affected bone is not as strong as normal bones and may fracture with minor trauma. There are no systemic symptoms. Most commonly affected bones are the metaphyses of long tubular bones (femur, tibia or humerus). The classic finding on X-ray is a "sunburst" pattern due to the aggressive lytic bones lesions that form that are associated with periosteal reaction. MRI is the most sensitive test to determine the extent of the tumor, but a tissue sample is required to confirm the diagnosis. Micrometastases are common at presentation so treatment often includes surgical resection of the tumor and chemotherapy. Ewing sarcoma (A) is less common and usually affects the diaphyses of long bones. It has a classic "onion skinning" pattern on X-ray. Osgood-Schlatter disease (B) is due to microfractures of the tibial tubercle from running and jumping activities, often in adolescents. Pain is worse with activity and often is so bad that activities must be stopped. Patellar tendonitis (D) is an overuse injury also caused by running or jumping. It is characterized by tenderness over the patellar tendon and occurs with activity. Imaging will be normal.

A patient was treated for an upper respiratory tract infection 2 weeks ago. She now presents with one day of 7/10 facial pain with bending forward, as well as difficulty blowing her runny nose. Examination reveals frontal bone tenderness to percussion and nasal erythema and drainage. Her temperature is 100°F. Which of the following is the most appropriate management option for this patient? Antihistamines Computed tomography of the frontal sinuses Decongestants Otolaryngological referral

Correct Answer ( C ) Explanation: Rhinosinusitis is one of the most common reasons a patient seeks medical care. Inflammation of the sinus epithelium almost never occurs without inflammation of the nasal epithelium, as such, rhinosinusitis is a more correct term. Onset can be acute, subacute, chronic and recurrent acute. Acute cases can be further defined as viral or bacterial, however, most cases are due to associated viral upper respiratory infection (the common cold). Symptoms include an onset after an upper respiratory tract infection, facial pain or pressure, nasal obstruction, rhinorrhea (may be purulent) and maxillary toothache. Symptoms lasting less than 7-10 days points toward a viral etiology. If symptoms last longer than 7 days, consider a bacterial etiology. Treatment begins with analgesics, decongestants and saline nasal irrigation. If the presenting temperature is greater than 101°F, or symptoms are severe or worsening or last longer than 7 days, begin antibiotics like amoxicillin or trimethoprim-sulfamethoxazole. Computed tomography of the sinuses (B) is not considered routine evaluation. It is reserved in cases of recurrent acute or chronic presentation, and is used to define suspected anatomic abnormalities. Antihistamines (A) are not recommended for the treatment of acute rhinosinusitis. Referral to an otolaryngologist (D) is recommended if patients fail medication therapy or CT scanning reveals sinus abnormalities.

A febrile infant with first urinary tract infection should be referred to undergo what other evaluation? Intravenous pyelogram Nuclear scanning with technetium-labeled dimercaptosuccinic acid (tc-99m DMSA scan) Renal and bladder ultrasonography Voiding cystourethrography

Correct Answer ( C ) Explanation: The 2011 AAP UTI Clinical Practice Guideline, Action Statement 5, states that febrile infants with 1st UTI should undergo renal and bladder ultrasonography (RBUS) to detect anatomic abnormalities that may require further evaluation and intervention. The RBUS should be done during the first 2 days of treatment to identify serious complications such as renal or perirenal abscesses or pylonephrosis if the patient's clinical illness is unusually severe or is not clinically improving. However, if the patient demonstrates significant improvement, then the RBUS should be performed after resolution of the acute illness. Intravenous pyelogram (A) can identify the patient's anatomy and some information on the functioning of the renal system. However, the study unnecessarily exposes the infant to intravenous contrast and radiation. Renal and bladder ultrasonography is a safer alternative. Tc-99m DMSA scan (B) has greater sensitivity for detection of acute pyelonephritis than RBUS or voiding cystourethrogram (VCUG), but is not recommended as routine evaluation in infants with 1st febrile UTI because of the radiation dose that may increase with follow-up studies and reduced renal function. VCUG (D) should not be performed routinely after 1st febrile UTI due to its cost and radiation exposure. Also the benefit of identifying high-grade vesicoureteral reflux is unclear, given that the treatment with antimicrobial prophylaxis seems to be ineffective in preventing recurrence of febrile UTIs.

A 59-year-old man presents to the ED with 12 hours of emesis and abdominal pain. Vital signs are T 38.2°C, BP 110/79, and HR 109. On exam, you note a tender 2-x-2-cm bulge with erythema in the abdominal midline above the umbilicus. There is abdominal distension, and an occasional high-pitched bowel sound is heard. After placing an IV line and nasogastric tube, which of the following is the most appropriate course of management? Administer broad-spectrum antibiotics and attempt reduction Administer broad-spectrum antibiotics and obtain a plain radiograph Administer broad-spectrum antibiotics and prepare the patient for the OR Administer broad-spectrum antibiotics then obtain a CT scan of abdomen

Correct Answer ( C ) Explanation: The clinical scenario is consistent with a strangulated loop of bowel (emesis, abdominal pain, fever, erythema, abdominal distension, high-pitched bowel sounds) incarcerated in a ventral hernia. If the contents of a hernia can be returned to their natural cavity by manual reduction, the hernia is termed reducible; if they cannot, it is termed irreducible or incarcerated. Incarcerated hernias (third most common cause of SBO) are subject to inflammatory and edematous changes and are at risk for strangulation, which refers to vascular compromise of the incarcerated contents. When strangulation is not emergently relieved, necrosis and gangrene develop. The treatment for an incarcerated hernia that cannot be manually reduced is surgical fixation. If strangulation is suspected or shock is present, broad-spectrum antibiotics and fluid resuscitation are necessary, but manual reduction is contraindicated because the patient requires surgical management. The reintroduction of ischemic, necrotic bowel back into the peritoneal cavity can result in subsequent perforation and sepsis. If the hernia is incarcerated, but the patient does not yet show signs of strangulation, then an attempt at reduction should be made in the ED. Antibiotics should be administered, but the patient requires surgical intervention for the strangulated loop of bowel rather than manual reduction (A), radiograph (B), or CT scan (D).

A 12-year-old girl presents for her annual well child check. She had menarche eight months prior, and has had menses every month. Each cycle lasts two weeks. The patient uses at least six pads on most days of the cycle. On exam, she is pale but otherwise well appearing. Her heart rate is 80 beats per minute, blood pressure 110/65, and respiratory rate 14. The remainder of her exam is within normal limits. Which of the following are the most likely laboratory findings? Hemoglobin 12 g/dL, mean corpuscular volume 80, RDW 10 Hemoglobin 6 g/dL, mean corpuscular volume 60, RDW 12 Hemoglobin 6 g/dL, mean corpuscular volume 60, RDW 18 Hemoglobin 8 g/dL, mean corpuscular volume 100, RDW 10

Correct Answer ( C ) Explanation: This patient has iron deficiency anemia secondary to dysfunctional uterine bleeding. Iron deficiency anemia is characterized by a microcytic anemia (low hemoglobin and low MCV). Approximately two percent of adolescent females experience iron deficiency anemia due to menstrual blood loss and the adolescent growth spurt. Most children with iron deficiency anemia are asymptomatic. The most common sign is pallor, which usually does not occur until the hemoglobin level is < 8 g/dL. Iron studies in this patient show reduced iron stores (low ferritin), increased iron-binding capacity (high transferrin and elevated TIBC), and decreased transferrin saturation. The mean corpuscular volume (MCV) describes red blood cells as microcytic, normocytic, or macrocytic. A normal MCV in an adolescent female is approximately 80. The red cell distribution width (RDW) is a measure of the variability in size of red blood cells. Iron deficiency anemia is characterized by an elevated RDW (11.5-14.5). A normocytic anemia with normal RDW (B) may be seen in alpha or beta thalassemia. These diagnoses also are associated with an elevated red blood cell count. Macrocytic anemias (D) are primarily cause by folate or vitamin B12 deficiency. Normal lab values (A) would be unlikely to occur in a patient with dysfunctional uterine bleeding and pallor.

A 53-year-old woman with chronic alcohol abuse presents with confusion and blurred vision. Her vital signs are normal. Physical examination reveals a wide based gait, inability to abduct her right eye fully, nystagmus and difficulty with memory. Her alcohol level is 0 mg/dl. Which of the following is most likely to diagnose this patient's disease? Administration of tensilon Non-contrast Head CT Response to thiamine administration Serum magnesium

Correct Answer ( C ) Explanation: This patient presents with Wernicke syndrome, a clinical diagnosis that can be corroborated by improvement of symptoms after thiamine administration. Wernicke's encephalopathy is a medical emergency with a high mortality rate (10-20%). Unfortunately, it often goes unrecognized. Diagnosis requires two of the following criteria: 1) dietary deficiency, 2) oculomotor abnormalities, 3) cerebellar dysfunction and 4) altered mental status or memory impairment. Oculomotor findings include nystagmus and ophthalmoplegia. Cerebellar signs typically manifest as ataxia. Patients with chronic malnourishment including alcoholics, patients with eating disorders and patients with advanced cancer are at risk for Wernicke's. Emergent treatment consists of thiamine administration. Ophthalmoplegia and nystagmus may resolve in hours and confirms diagnosis. Tensilon (A) is used for the diagnosis of myasthenia gravis, which typically presents with ptosis, blurred vision and diplopia that worsens during the day. A non-contrast head CT (B) may show signs of volume loss secondary to alcohol abuse but will not aid in diagnosis or management. Although many alcoholics are hypomagnesemic (D) this patient's symptoms are not a result of low magnesium.

Which of the following tick-borne illnesses is most associated with skin ulcers and lymphadenopathy? Babesiosis Colorado tick fever Lyme disease Tularemia

Correct Answer ( D ) Explanation: The major tick-borne diseases include Rocky Mountain spotted fever (RMSF), Lyme disease, ehrlichiosis, babesiosis and tularemia. Tularemia is caused by Francisella tularensis and is transmitted to humans either by tick bites or handling of infected animals (rabbits and rodents). Each of these diseases begins with non-specific viral-syndrome like symptoms including fever, myalgias, arthralgias and headache. Clinicians may be alerted to the presence of a tick-borne illness if a history of exposure (hiking in the woods, tick bite) are elicited. However, up to 50% of patients with a tick-borne illness do not recall a tick bite. Additionally, if labs are obtained, there are some common findings that would differentiate from a viral illness. Thrombocytopenia is often seen in RMSF, ehrlichiosis, tularemia and babesiosis. A mild elevation in hepatic transaminases is associated with RMSF, Lyme disease, ehrlichiosis, tularemia and babesiosis. Tularemia is associated with ulceroglandular disease. Ulceroglandular disease is not typically seen in Lyme disease (C), Colorado tick fever (B) or babesiosis (A).

A 15-year-old boy is sent to you by the athletic trainer of the local football team after suffering his 3rd concussion of the season. Based on current guidelines, your advice pertaining to this patient returning to play should be which of the following? He may return to play if asymptomatic after 2 weeks He may return to play if asymptomatic after 30 days He may return to play that day if exam results are normal at rest and with exertion He should terminate play for the season

Correct Answer ( D ) Explanation: A concussion, also known as mild traumatic brain injury (MTBI), is defined as an alteration in cerebral function secondary to a direct or indirect force on the brain. Current guidelines state that after an athlete's 3rd concussion, he or she should terminate the current season but may return to play the subsequent season if asymptomatic. An athlete can return to play if asymptomatic after 2 weeks (A) after his or her 1st concussion (Grade 2). An athlete can return to play if asymptomatic after 1 month (B) after his or her 1st concussion (Grade 3) or 2nd concussion (Grade 2). An athlete can return to play if asymptomatic the same day (C) after his or her 1st concussion (Grade 1).

A previously healthy 48-year-old man presents with fever, headache and signs of meningeal irritation. He has no significant past medical history. He has no rash and has not had any recent medical procedures. Which of the following is the most common bacteria responsible for the suspected diagnosis? Haemophilus influenzae Neisseria meningitidis Staphylococcus aureus Streptococcus pneumoniae

Correct Answer ( D ) Explanation: Adult bacterial meningitis has several microbiologic etiologies. The most common cause is Streptococcus pneumoniae (30-60% of cases) for all age categories. The second most common bacteria is Neisseria meningitidis (10-35%), which more commonly affects older children and young adults, and more commonly occurs is associated with a petechial rash. Haemophilus influenzae is also causative (<5%), and occurs most commonly in those with recent neurosurgical procedures or possible CSF leaks. Listeria monocytogenes (5-10%) and Gram-negative rods (1-10%) are also causative and occur more frequently in elderly patients or those with alcoholism, immunosuppression, cancer and recent medical or neurosurgical procedures. Empiric treatment in healthy adults is ceftriaxone plus vancomycin. Consider ampicillin if the patient is older than 50 years or has an alcohol abuse history in order to cover Listeria monocytogenes. Haemophilus influenzae (A) is not a common source in this age group nor in those without recent neurosurgical procedures. Neisseria meningitidis (B) is less common in this age group, and is less common overall when compared to Streptococcus pneumoniae. Staphylococcus aureus (C) is seen in patients with indwelling catheters, recent head trauma and recent neurosurgical procedures.

Which of the following statements is true regarding febrile seizures? Administering acetaminophen and ibuprofen during a febrile illness have been shown to decrease the likelihood of seizure recurrence Children who have had a simple febrile seizure have the same rate of epilepsy as those who have not had a febrile seizure Older children with a febrile seizure are more likely to have a recurrence than younger children with a febrile seizure are Treatment with long-term anticonvulsants does not lower the long-term risk of developing epilepsy

Correct Answer ( D ) Explanation: Children with simple febrile seizures have a 2% to 3% chance of developing epilepsy, compared with a 1% rate of epilepsy in the general population. Children with complex febrile seizures have a significantly higher risk. Treatment with long-term anticonvulsants does not affect the long-term risk of developing epilepsy and is rarely warranted. Administering acetaminophen and ibuprofen (A) have not been shown to decrease the likelihood of seizure recurrence. About 30% of children with a simple febrile seizure have a recurrence; of these, one-half will have a third event. Children with simple febrile seizures have a 2% to 3% chance of developing epilepsy (B), compared with a 1% rate of epilepsy in the general population. The younger the age at onset of a febrile seizure, the more likelihood of recurrence (C).

A patient with schizophrenia is starting treatment with clozapine. Which of the following needs to be monitored weekly? Electrocardiogram Fasting plasma glucose Hemoglobin and hematocrit White blood cell and absolute neutrophil count

Correct Answer ( D ) Explanation: Clozapine is a second-generation antipsychotic used in the treatment of schizophrenia that is found resistant to other antipsychotics. It may also be helpful in treating individuals with schizophrenia who exhibit self-injurious or suicidal-type behaviors. Clozapine has a unique side effect profile and patients prescribed this medication are entered into a computer-based registry that requires specific monitoring parameters during the course of treatment. Use of clozapine involves a risk of life-threatening agranulocytosis, therefore regular monitoring of white blood cell and absolute neutrophil count (ANC) is required. A baseline complete blood count and ANC is performed prior to initiation of therapy, then done weekly for the first six months of therapy. Other adverse effects of clozapine may include myocarditis, pulmonary embolism, weight gain, insulin resistance, seizures, and sedation. Because of the potential risks, patients determined to be candidates for treatment with clozapine should be referred to a psychiatric provider with experience in treating patients with this agent. Due to the risk of clozapine-induced myocarditis, an electrocardiogram (A) should be obtained at baseline prior to initiating treatment, then weekly monitoring of eosinophil count, troponin, and sedimentation rate or C-reactive protein needs to occur for at least the first four weeks of treatment. Metabolic side effects including hyperglycemia, insulin resistance and diabetes mellitus can occur with clozapine. Fasting plasma glucose (B) is recommended monthly at the initiation of treatment. Screening with hemoglobin A1C may also be used. Monitoring of hemoglobin and hematocrit (C) does not play a role in the management of patients taking clozapine.

Which of the following characteristics is associated with Crohn's Disease? Continuous uninterrupted inflammation of the colonic mucosa Increased risk for the development of colorectal carcinoma Smoking appears to confer a protective effect Transmural involvement with cobblestone appearance

Correct Answer ( D ) Explanation: Crohn's disease is a chronic, transmural inflammatory disease of the gastrointestinal tract for which the cause is unknown. With disease progression, ulceration results in complete transmural inflammation.The ulcers are characteristically linear and may coalesce to produce transverse sinuses separated by islands of normal mucosa, thus giving the characteristic cobblestone appearance. Crohn's disease can involve any part of the alimentary tract from the mouth to the anus but most commonly affects the small intestine and colon. The most common clinical manifestations are abdominal pain, diarrhea, and weight loss. Crohn's disease can be complicated by intestinal obstruction or localized perforation with fistula formation. Medical and surgical treatments are palliative; however, operative therapy can provide effective symptomatic relief for patients with complications from Crohn's disease and produces a reasonable long-term benefit. Thirty percent of patients present with small bowel disease alone and, in 15%, the disease appears limited to the large intestine. The disease process is discontinuous and segmental. In patients with colonic disease, rectal sparing is characteristic of Crohn's disease and helps distinguish it from ulcerative colitis. Perirectal and perianal involvement occurs in about one third of patients with Crohn's disease evidenced by the development of aphthous ulcers, anal fissues, perirectal abscesses, and anorectal fistuals. Ulcerative colitis is associated with continuous uninterrupted inflammation of the colonic mucosa (A), an increased risk of the development of colorectal carcinoma (B), and tobacco use (C) appears to confer a protective effect.

A patient with sarcoidosis presents with progressive peripheral edema. Echocardiography reveals restrictive cardiomyopathy. In addition to diuresis and heart rate control, which of the following is also appropriate? Buspirone Melphalan Phlebotomy Prednisone

Correct Answer ( D ) Explanation: Infiltrative myocardial deposition usually results in restrictive cardiomyopathy (RCM). Evaluation begins with the typical heart failure work-up of chest radiograph, ECG, echocardiogram, cardiac MRI and possibly cardiac catheterization. A chest radiograph may show enlarged atria and normal ventricles. ECG may reveal low voltages and tachyarrhythmias. Echocardiographic findings typical of RCM include bilateral atrial enlargement, normal or thickened ventricular walls, decreased right > left ventricular walls, mural thrombi and diastolic dysfunction. A cardiac MRI may show myocardial inflammation or infiltrates. Endocardial biopsy may be necessary to confirm a diagnosis. Treatment of RCM is difficult due to the relative refractoriness of many of the underlying pathologies. Specific augmentative treatments are aimed at the underlying disorder. One example is corticosteroids, like prednisone or prednisolone, used to control sarcoidosis and Loeffler's endocarditis. Gentle diuresis may be helpful. Anticoagulation is usually considered. Serotonin agonists are contraindicated in patients with restrictive cardiomyopathy, as these drugs, like buspirone (A), cisapride and sumatriptan, can cause decreased compliance of the endomyocardium, and actually lead to restrictive cardiomyopathy. Chemotherapy, like melphalan (B) and cyclophosphamide, is used to decrease the amount of abnormal cells in amyloid restrictive cardiomyopathy. Phlebotomy (C) and chelation therapy are used to control the complications of hemochromatosis.

A woman complains of her heart racing, facial flushing and headache after taking an unknown pill in her purse. Which of the following is the most likely medication? Amitriptyline Metformin Metoprolol Nitroglycerin

Correct Answer ( D ) Explanation: Nitroglycerin can cause reflex tachycardia, flushing and headache. It produces a vasodilator effect on the peripheral veins and arteries with more prominent effects on the veins. Nitroglycerine primarily reduces cardiac oxygen demand by decreasing preload as well as some reduction in afterload. It dilates coronary arteries and improves collateral flow to ischemic regions. Nitroglycerine vasodilates blood vessels by releasing nitric oxide in smooth muscle. This vasodilation can cause hypotension. The elevated heart rate is reflexive in nature; the heart attempts to compensate for the drop in blood pressure. Metformin (B) is a common medication used in the treatment of type two diabetes mellitus. The most common side effects are gastrointestinal, including a metallic taste, mild anorexia, nausea, abdominal discomfort and diarrhea. Metoprolol (C) is a beta blocker with many cardiovascular indications. Depression, fatigue, and sexual dysfunction are commonly cited side effects of beta blockers. Amitriptyline (A) is a tricyclic antidepressant. Tricyclics block muscarinic M1, histamine H1, and alpha-adrenergic receptors, and commonly cause cardiac effects, anticholinergic effects, antihistaminic effects, decreased seizure threshold, sexual dysfunction, diaphoresis, and tremor.

A 65-year-old patient is diagnosed with stage III endometrial adenocarcinoma. You decide to refer her to a gynecologic oncologist. She asks you to explain the "best treatment available." In your counseling, you discuss which of the following treatment options? Chemotherapy External beam radiation Partial vaginal hysterectomy Total abdominal hysterectomy

Correct Answer ( D ) Explanation: Primary surgical excision is the main treatment of endometrial carcinoma and involves opening the abdominopelvic cavity and obtaining peritoneal washings, followed by total hysterectomy with bilateral salpingo-oophorectomy. The decision to include pelvic, periaortic and inguinal lymph node sampling is based on the presence and amount of myometrial invasion. Chemotherapy (A) with doxorubicin and cisplatin is associated with poor long-term remission. Recurrence rates are significantly decreased with the use of postoperative external beam radiation, while patient's with significant tumor bulk benefit from preoperative radiation. However, any radiation (B) is considered adjuvant therapy to surgical excision. Vaginal hysterectomy (C) is reserved for patients with stage I disease, especially if they cannot tolerate the rigors of abdominal surgery. One Step Further Question: What is the first-line therapy for recurrent endometrial carcinoma? Reveal Answer: Rapid Review Endometrial Cancer Patient will be a postmenopausal woman Complaining of abnormal vaginal bleeding Diagnosis is made by transvaginal ultrasound or endometrial biopsy Most common type is adenocarcinoma Treatment is total abdominal hysterectomy and bilateral salpingo-oophorectomy (TAH-BSO)

A 30-year-old woman presents to her family physician for her annual exam. Her last Papanicolaou test with HPV co-testing was 4 years ago. She has 2 children and underwent a bilateral tubal ligation with her last cesarean section. She does not smoke or have any significant past medical history. Her mother died of breast cancer at age 45. Her aunt was treated with chemotherapy and surgery for ovarian cancer. Which of the following prevention measures or diagnostic studies should this patient initially undergo? BRCA screening Genetic counseling National Cancer Institute Breast Cancer Risk Assessment Tool Ontario Family History Assessment Tool

Correct Answer ( D ) Explanation: The cancer types related to potentially harmful mutations of the BRCA genes are predominantly breast, ovarian, and fallopian tube cancer, although other types are also associated. The U.S. Preventive Services Task Force (USPSTF) recommends that primary care clinicians screen women who have family members with breast, ovarian, tubal, or peritoneal cancer with one of several screening tools designed to identify a family history that may be associated with an increased risk of potentially harmful mutations in breast cancer susceptibility genes (BRCA1 or BRCA2). Women with positive screening results should receive genetic counseling and, if indicated after counseling, BRCA testing. Although several risk tools are available, the tools evaluated by the USPSTF include the Ontario Family History Assessment Tool, Manchester Scoring System, Referral Screening Tool, Pedigree Assessment Tool, and FHS-7. In general, these tools elicit information about factors that are associated with increased likelihood of BRCA mutations. Family history factors associated with increased likelihood of potentially harmful BRCA mutations include breast cancer diagnosis before 50 years of age, bilateral breast cancer, presence of breast and ovarian cancer, presence of breast cancer in one or more male family members, multiple cases of breast cancer in the family, one or more family members with two primary types of BRCA-related cancer, and Ashkenazi Jewish ethnicity. Screening tools should be implemented initially before genetic counseling (B) and BRCA1 testing (A). National Cancer Institute Breast Cancer Risk Assessment Tool (C) is based on the Gail model and should not be used as a screening tool to determine which patients would benefit for BRCA risk assessment or the genetic counseling that precedes the testing.

A 3-year-old boy presents with left anterior cervical lymphadenitis and fever that developed over three days. The area surrounding the node is tender, warm, and swollen. What is the most likely etiology of the lymphadenitis? Atypical mycobacteria Bartonella henselae Epstein-Barr Virus Staphylococcus aureus

Correct Answer ( D ) Explanation: The most common causes of infectious lymphadenitis in children are Staphylococcus aureus and Streptococcus pyogenes. These two pathogens alone cause up to 80% of cervical lymphadenitis in children < 5 years old. Staph and Strep species cannot be differentiated by clinical presentation alone. Thus, appropriate antimicrobial therapy should cover both Strep and Staph species, including methicillin-resistant Staph aureus. Patients who fail to show improvement after 2-3 days of appropriate antibiotics should be evaluated for the development of fluctuance, which indicates a developing abscess. Atypical mycobacteria (A) are a common cause of lymphadenitis. In comparison to the acute onset of Staph or Strep lymphadenitis, the presentation of atypical mycobacterial lymphadenitis is subacute. The involved nodes are classically non-tender and may develop overlying skin changes and draining sinus tracts. The most common location of atypical mycobacterial lymphadenitis is submandibular. Bartonella henselae (B) is the cause of Cat Scratch Disease, which commonly causes axillary or cervical lymphadenitis in children. History includes exposure to kittens, cats, or fleas within the prior weeks. Often an erythematous papule can be found at the inoculation site for several weeks after acquisition of Bartonella. Epstein-Barr Virus (C) mononucleosis is a common cause of fever and lymphadenopathy in children and adolescents. The lymphadenopathy associated with EBV typically is bilateral, involves the posterior cervical lymph nodes, and does not result in overlying skin changes. Sore throat is a common clinical complaint, and exudative pharyngitis is often noted on exam. The lymphadenopathy caused by EBV may also be generalized and may be accompanied by splenomegaly or hepatomegaly.

A 47-year-old woman with a history of human immunodeficiency virus infection, hypertension, and hyperlipidemia, presents with a 1-day history of epigastric abdominal pain, nausea and vomiting. Vital signs are notable for BP 135/86 mm Hg, pulse of 104 beats/minute, RR 14 breaths/minute, temperature of 37.9℃, and oxygen saturation of 98% on room air. On examination, she is exquisitely tender in the epigastrium. Lipase is elevated at 4,500 U/L. Which of the following of her medications is the most likely cause of her symptoms? Atorvastatin Indinavir Lisinopril Tenofovir

Correct Answer ( D ) Explanation: The patient has clinical and laboratory evidence of acute pancreatitis. Common causes of pancreatitis include alcohol, gallstones, and medications. In particular, the nucleoside reverse transcriptase inhibitors used in the treatment of HIV are commonly associated with pancreatitis. Common culprits include tenofovir, didanosine, and stavudine. Treatment of medication-induced pancreatitis includes supportive care, bowel rest, and cessation of the offending agent. Atorvastatin (A) is an HMG-CoA reductase inhibitor, commonly known as a statin, used in the treatment of hyperlipidemia. Common adverse effects of statins include myopathy and rhabdomyolysis. Indinavir (B) is a protease inhibitor used in the treatment of HIV. Nephrolithiasis is a well-recognized complication of indinavir use. Lisinopril (C) is an antihypertensive medication in the ace-inhibitors class. Important adverse effects of ace-inhibitors include cough and angioedema.

A 47-year-old man with chronic low back pain presents with epigastric pain for 3 weeks. The pain is burning, without radiation and occurs 2-3 hours after eating. Vital signs are normal. He takes ibuprofen and naproxen for his back pain. What management is indicated? Admit for endoscopy Start esomeprazole and refer for outpatient evaluation Start ranitidine and refer for outpatient evaluation Stop ibuprofen and naproxen and refer for outpatient evaluation

Correct Answer ( D ) Explanation: The patient presents with clinical gastritis likely secondary to the use of non-steroidal anti-inflammatory drugs (NSAIDs). Technically, gastritis is a histologic diagnosis indicating inflammation of the gastric mucosa and can only be definitively made after endoscopy and biopsy. Clinicians, however, refer to the symptoms of dyspepsia (pain in the upper abdomen that presents as bloating or heartburn) as gastritis. The most common cause of gastritis is Helicobacter pylori infection but it also commonly results from medications including salicylates and NSAIDs (ibuprofen, naproxen etc.). Acute gastritis typically presents with abdominal pain located in the epigastric area, which is burning in nature and can also have bloating or nausea associated with it. Treatment should begin with removal of any possible inciting agents including alcohol, smoking, steroids or NSAIDs. Ranitidine (C) is a histamine H2 receptor antagonist and esomeprazole (B) is a proton-pump inhibitor (PPI). Both of these drugs act by reducing acid production in the stomach and can reduce dyspepsia. However, these drugs should be second line to removing any inciting agents. Endoscopy (A) can be performed to definitively diagnose gastritis and to rule out other causes of symptoms (peptic ulcer disease, gastric tumor etc.). However, endoscopy is not indicated in the emergent setting for mild to moderate symptoms. Referral to a gastroenterologist for consideration of an upper endoscopy is reasonable.

A 26-year-old man presents with a 2-week history of fever and a cough. He was diagnosed with HIV four months ago and is not on any antiretroviral medications. His vital signs are BP 122/76, HR 78, RR 16, oxygen saturation 92% on room air, and temperature 99.2°F. Chest X-ray demonstrates diffuse interstitial infiltrates bilaterally. Given this presentation, it is suspected that he has pneumonia secondary to Pneumocystitis jiroveci. Which of the following antibiotics is used to treat suspected pneumonia caused by Pneumocystitis jiroveci? Azithromycin Clindamycin Penicillin Trimethoprim-sulfamethoxazole

Correct Answer ( D ) Explanation: This patient has pneumonia secondary to Pneumocystitis jiroveci (PCP pneumonia). PCP generally occurs in immunocompromised patients, including HIV patients with a CD4 count less than 200 cells/mL. Once a patient's CD4 count is less than 200 cells/mL, daily prophylaxis should be initiated with trimethoprim-sulfamethoxazole to prevent PCP. Patients with PCP often present with a cough, unexplained fever, shortness of breath, and hypoxia out of proportion to the clinical picture. Chest X-ray classically shows diffuse interstitial infiltrates, but a negative chest X-ray can be seen in up to 20% of patients. Since the organism cannot be grown in the laboratory, diagnosis depends on other means. Most often, diagnosis is confirmed through bronchoscopy with a bronchoalvelolar lavage. A classic finding in patients with PCP is an elevated lactate dehydrogenase. Treatment, which should be initiated upon early suspicion and before diagnosis, is with trimethoprim-sulfamethoxazole. In addition, patients who are hypoxic with a PaO2 of less than 70 mm Hg (equivalent to an oxygen saturation of <~93%) or have an elevated alveolar-arterial gradient of greater than 35 mm Hg, should receive steroids prior to initiation of antibiotics. Azithromycin (A) is a macrolide antibiotic commonly prescribed for community-acquired pneumonia. Clindamycin (B) is generally given for aspiration pneumonia. Penicillin (C) is infrequently prescribed due to high resistance, but it is remains the first line treatment for syphilis.

A 54-year-old man with chronic alcohol abuse presents with confusion and visual hallucinations. Vital signs are HR 113, BP 164/95, finger stick blood glucose 113, and T 100.5°F. Physical examination reveals a disheveled man with tremors, tongue fasciculations, and agitation. What management should be pursued? Admission to psychiatry for management Cyproheptadine Dantrolene Diazepam

Correct Answer ( D ) Explanation: This patient presents with alcohol withdrawal syndrome (AWS) complicated by delirium tremens and requires treatment with benzodiazepines (diazepam). AWS is a complicated disease that is not fully understood. Chronic alcohol consumption causes depression of the central nervous system (CNS). Removal of the sedative leads to uncontrolled CNS excitation represented by autonomic hyperactivity and altered mental status. Patients may exhibit tachycardia, hypertension, hyperthermia, anxiety, tremors, tongue fasciculations, hyper-reflexia, sleep disturbance, auditory and visual hallucinations and seizures. In delerium tremens, patients will have gross tremors, marked visual hallucinations, profound confusion and agitation. Withdrawal symptoms can begin within 6 hours of cessation of alcohol intake and typically peaks after 24-36 hours of abstinence. It is vital to investigate the cause of alcohol cessation as it may be due to another severe medical illness (pneumonia, meningitis, sepsis, myocardial infarction, cerebrovascular accident etc.). Treatment focuses on supportive care and sedation with benzodiazepines. Some patients may be resistant to benzodiazepines either due to cross-tolerance or concomitant benzodiazepines abuse. Patients refractory to these medications can be treated with barbiturates (typically phenobarbital) and in severe cases, propofol. A primary psychiatric diagnosis (A) should not be made in a chronic alcoholic with abnormal vital signs. Cyproheptadine (B) is the antidote for serotonin syndrome, which can present with autonomic instability but should also have muscle rigidity and clonus. Dantrolene (C) is the antidote for patients suffering from neuroleptic malignant syndrome.


Related study sets

Social Studies: American Revolution pg. 228-231 (#19-24)

View Set

Business Data Networks & Security - Ch. 6, Business Data Networks and Security - Chapters 8, 9, 10, and 11., Business Data Networks & Security (Panko)- Ch 9- Internetworking, Business Data Networks and Security Ch 10, Business Data Networks & Securit...

View Set

Fundamentals of Criminal Law Chapter 2

View Set

PREPU (UNFINISHED) Chapter 18: CNS Stimulants

View Set